You are on page 1of 143

Engineering Mathematics

Complex Variables
Spring Semester, 2016

Textbook
Ch. 17 ~ 20, Advanced Engineering Mathematics, by D. G. Zill, Warren
S. Wright, and M. R. Cullen, 5th edition, Jones and Bartlett Publishers

Reference Books
Complex Variables and Applications, by James W. Brown and Ruel V.
Churchill, 6th edition, McGraw-Hill, Inc.
A First Course in Complex Analysis with Applications, by Dennis. G.
Zill and Patrick D. Shannahan, 2nd edition, Jones and Bartlett
Publishers

Grading Policy
Midterm: 40%
Final: 40%
Quiz: 20%
2

Contents
Functions of a Complex Variable
Integration in the Complex Plane
Series and Residues
Conformal Mappings

Chapter 17
Functions of a Complex Variable

Contents
17.1 Complex Numbers
17.2 Powers and Roots
17.3 Sets of points in the Complex Plane
17.4 Functions of a Complex Variable
17.5 Cauchy-Riemann Equations
17.6 Exponential and Logarithmic Functions
17.7 Trigonometric and Hyperbolic Functions
17.8 Inverse Trigonometric and Hyperbolic Functions

A Definition
DEFINITION 17.1
Complex Number
A complex number is any number of the z = a + ib where a and b are
real numbers and i is the imaginary units.
Complex conjugate of z: z = a ib
Imaginary unit: i 2 = 1
Real part of z: a = Re ( z ) =

z+z
2

Imaginary part of z: b = Im ( z ) =

zz
2i

DEFINITION 17.2
Equality
Complex numbers z1 = x1 + iy1 and z2 = x2 + iy2 are equal, z1 = z2, if
Re(z1) = Re(z2) and Im(z1) = Im(z2).
z = x + iy = 0

x=0 &y=0

Arithmetic Operation
z1 = x1 + iy1

z2 = x2 + iy2

Addition and Subtraction


z1 z2 = ( x1 x2 ) + i ( y1 y2 )

z1 z2 = ( x1 + iy1 ) ( x2 + iy2 )

= ( x1 x2 ) + i ( y1 y2 )

z1 z2 = ( x1 x2 ) + i ( y1 y2 )

z1 z2 = z1 z2

= ( x1 x2 ) i ( y1 y2 )
= ( x1 iy1 ) ( x2 iy2 )
= z1 z2

Multiplication

z1 z2 = ( x1 + iy1 )( x2 + iy2 )

z1 z2 = ( x1 x2 y1 y2 ) + i ( y1 x2 + x1 y2 )

z1 z1 = x + y
2
1

z1 z2 = z1 z2

2
1

= ( x1 x2 y1 y2 ) + i ( y1 x2 + x1 y2 )

z1 z1 = ( x1 + iy1 )( x1 iy1 )
= x12 + y12

z1 z2 = ( x1 x2 y1 y2 ) + i ( y1 x2 + x1 y2 )

= ( x1 x2 y1 y2 ) i ( y1 x2 + x1 y2 )
= ( x1 iy1 )( x2 iy2 ) = z1 z2

Division
z1 ( x1 x2 + y1 y2 ) + i ( y1 x2 y2 x1 )
=
z2
x22 + y22
z1
z
= 1
z2
z2

Commutative Laws
z1 + z2 = z2 + z1
z1 z2 = z2 z1

z1 z1 z2
=
z2 z2 z2

( x1 + iy1 )( x2 iy2 )
( x2 + iy2 )( x2 iy2 )
( x x + y y ) + i ( y1 x2 y2 x1 )
= 1 2 1 2
=

x22 + y22

z1
=
z2
=
=

Associative Laws
z1 + ( z2 + z3 ) = ( z1 + z2 ) + z3

z1 ( z2 z3 ) = ( z1 z2 ) z3

( x1 x2 + y1 y2 ) + i ( y1 x2 y2 x1 )
x22 + y22

( x1 x2 + y1 y2 ) i ( y1 x2 y2 x1 )
z2 z2

x2 ( x1 iy1 ) + iy2 ( x1 iy1 )


z2 z 2

( x1 iy1 )( x2 + iy2 ) =
z2 z2

z1 z2 z1
=
z2 z2 z2

Distributive Laws
z1 ( z2 + z3 ) = z1 z2 + z1 z3
8

Geometric Interpretation
imaginary axis

A complex number
z = x + iy

ordered pair (x, y)


a point (x, y) in complex plane
(z-plane)
a vector x + iy

real axis

DEFINITION 17.3
Modulus or Absolute Value
The modulus or absolute value of z = x + iy, denoted by |z|, is the real
number
z = x 2 + y 2 = zz

Vector Sum
z1 + z2 = ( x1 + x2 ) + i ( y1 + y2 )

Triangle Inequality
z1 + z2 z1 + z2

z1 + z2 z1 z2

( z1 + z2 ) + ( z2 )

z1 + z2 + z2

z1 + z2 z1 z2

z1 + z2 + ...... + zn z1 + z2 + ...... + zn

We cannot compare two complex numbers


z1 = x1 + iy1 , y1 0
z2 = x2 + iy2 , y2 0

The geometric meaning of |z1 z2| is


the distance from the point z1 to the
point z2 on z-plane.

z1 > z2 and z1 z2 have no meaning


10

Contents
17.1 Complex Numbers
17.2 Powers and Roots
17.3 Sets in the Complex Plane
17.4 Functions of a Complex Variable
17.5 Cauchy-Riemann Equations
17.6 Exponential and Logarithmic Functions
17.7 Trigonometric and Hyperbolic Functions
17.8 Inverse Trigonometric and Hyperbolic Functions

11

Polar Form

z = x + iy

z = r ( cos + i sin ) = r

magnitude or modulus of z
r = z = x2 + y 2

argument of z
y
y
= arg z tan =
x
x
arg z = Arg z + 2n (n : any integer)

(in radius) = tan 1

principal argument of z
Arg z = 0 , < 0

A Complex Number in Polar Form

Example 1

Express 1 3i in polar form


r= z =

(1)

z = 1 3i = 2

z = 2 cos

+ 3

=2

1
3
5
5
i = 2 cos

+ i sin
2 2
3
3

+ i sin

Arg z =

arg z =

5
3
12

Multiplication and Division


z1 = r1 ( cos 1 + i sin 1 )

z2 = r2 ( cos 2 + i sin 2 )

z1 z2 = r1 ( cos 1 + i sin 1 ) r2 ( cos 2 + i sin 2 )


= r1r2
= r1r2

( cos 1 cos 2 sin 1 sin 2 ) + i ( sin 1 cos 2 + cos 1 sin 2 )


cos (1 + 2 ) + i sin (1 + 2 )

z1 z2 = r1r2 (1 + 2 )
z1 r1 ( cos 1 + i sin 1 )
=
z2 r2 ( cos 2 + i sin 2 )
=

r1 ( cos 1 + i sin 1 ) ( cos 2 i sin 2 )


r2 ( cos 2 + i sin 2 ) ( cos 2 i sin 2 )

r
= 1
r2
=

z2 0

z1 z2 = z1 z2
arg ( z1 z2 ) = arg z1 + arg z2
z
z1
= 1
z2
z2

( cos 1 cos 2 + sin 1 sin 2 ) + i ( sin 1 cos 2 sin 2 cos 1 )

r1
cos (1 2 ) + i sin (1 2 )
r2

arg

z1
= arg z1 arg z2
z2

z1 r1
= (1 2 )
z2 r2
13

Power of z
z = r ( cos + i sin )

z 2 = zz = rr cos ( + ) + i sin ( + ) = r 2 ( cos 2 + i sin 2 )


z1 z2 = r1r2 (1 + 2 )
z 3 = z 2 z = r 2 r cos ( 2 + ) + i sin ( 2 + ) = r 3 ( cos 3 + i sin 3 )
z 2 =

z0 1
1
= 2 cos ( 0 2 ) + i sin ( 0 2 ) = 2 cos ( 2 ) + i sin ( 2 )
2
z
r
r
z1 r1
= (1 2 )
z2 r2

z n = r n ( cos n + i sin n )

DeMoivres Formula
z = ( cos + i sin )

( cos + i sin )

z = r =1

= cos n + i sin n

14

Roots
A number w is said to be an nth root of a nonzero complex number z
wn = z
Let w = ( cos + i sin )

z = r ( cos + i sin )

wn = n ( cos n + i sin n ) = z = r ( cos + i sin )


1

n = r

=r n
+ 2k
=

n = + 2k

cos

+ 2 k
n

k = 0,1, 2,......, ( n 1)

2
n

k = 0,1, 2,......, ( n 1)

The nth roots of a nonzero complex number


z = r ( cos + i sin )
wk = r

w1

w2

+ i sin

w0

wn 1

wk

+ 2 k
n

n distinct roots (same modulus, different arguments)

Principal nth root of z: w0 = r

cos

+ i sin

15

Contents
17.1 Complex Numbers
17.2 Powers and Roots
17.3 Sets of Points in the Complex Plane
17.4 Functions of a Complex Variable
17.5 Cauchy-Riemann Equations
17.6 Exponential and Logarithmic Functions
17.7 Trigonometric and Hyperbolic Functions
17.8 Inverse Trigonometric and Hyperbolic Functions

16

Notes: Set Theory


z0 = x0 + iy0

z z0 =

( x x0 ) + ( y y0 )
2

C = z z z0 = : circle of radius centered at z0 , > 0

N = z z z0 < : -neighborhood of z0, or open disk

every z (in S) interior point

z z0 =

Notes:

Example

z0

N ( z0 ) S

z0 interior point of a set S


S open set

S1 = { z Re z > 1}

S2 = { z Re z 1}

S1: open set

S2: not open

17

Examples of open sets


S3 = { z Im z < 0}

S4 = { z 1 < Re z < 1}

S5 = z z > 1

S6 = z 1 < z < 2

18

S = z 1 < z z0 < 2 : open annulus

zb boundary point

every N ( zb ) contains at least one point S


and at least one point S

z0

B boundary of S

B contains all boundary points of S


Notes:

Example

S2 = { z Re z 1}

S7 = z z i 2

B2 = { z Re z = x = 1}

B7 = z z i = 2

Open set S connected


Any pair of z1 and z2 in S can be connected
by a polygonal line that lies entirely in S
S open connected set

set S domain

19

Examples

z2

z1

S3, S4, S5, S6 domains


S8 = { z Re z 4}

S8 open set
S8 connected

S8

S8

x4

region: domain with all, some or none of its boundary points


S closed region

Def: S is a closed set

S contains all its boundary points

S contains all its boundary points

Example

{
}
= { z 1 z 5 < 3}

S7 = z z i 2
S9

closed region, closed disk

include all boundary points

neither open nor closed


20

Contents
17.1 Complex Numbers
17.2 Powers and Roots
17.3 Sets in the Complex Plane
17.4 Functions of a Complex Variable
17.5 Cauchy-Riemann Equations
17.6 Exponential and Logarithmic Functions
17.7 Trigonometric and Hyperbolic Functions
17.8 Inverse Trigonometric and Hyperbolic Functions

21

Assign each element of set A to only one element of set B


function f : a A

b B

b = f (a)

b = f ( a ) : image of a under f

set A: domain of f

set R = {b b = f ( a ) , a A} : range of f
f function of a real variable x

A = set of real numbers

Functions of a Complex Variable


f function of a complex variable z or a complex function
A = set of complex numbers
w = f ( z ) = u ( x, y ) + iv ( x, y ): image of z under f

not a curve on x-y plane now!!

Example

f ( z ) = z 2 = ( x + iy ) = ( x 2 y 2 ) + 2ixy = u ( x, y ) + iv ( x, y )
2

u ( x, y ) = x 2 y 2
v ( x, y ) = 2 xy
22

Geometrical representation of complex function w = f (z):


as a mapping or transformation from z-plane to w-plane

w = u + iv

z = x + iy

23

Example 1

Image of a Vertical Line

Find the image of the line Re(z) = 1 under the mapping f (z) = z2
Sol:
x =1

w = f ( z ) = z 2 = u ( x, y ) + iv ( x, y )

u ( x, y ) = x 2 y 2 v ( x, y ) = 2 xy

Re ( z ) = 1

u = 1 y2

v = 2y

v2
u = 1
4

Note: w = f ( z ) = u ( x, y ) + iv ( x, y ) is
completely determined by u ( x, y )
and v ( x, y )

24

Complex Function as Flows


w = f(z) ~ two-dimensional fluid flow
~ a vector based at point z
f (z) specifies the speed and direction of the flow at a given point z
x(t ) + iy (t ) : parametric representation for the path of a particle in the flow

tangent vector T = x ' ( t ) + iy ' ( t )

f x ( t ) + iy ( t ) = f ( z ) = u ( x, y ) + iv ( x, y )

dx
= u ( x, y )
dt
dy
= v ( x, y )
dt

The solutions of this system: streamlines of the flow associated with f(z)

25

Example 2

Streamlines

Find the streamlines of the flows associated with the complex functions (a) f1 ( z ) = z
and (b) f 2 ( z ) = z 2
Sol:

(a) f1 ( z ) = z = x iy = u + iv
dx
=u = x
dt

dy
= v = y
dt

x ( t ) = c1et

y ( t ) = c2 et

x ( t ) y ( t ) = c1c2

hyperbola

f1 ( z ) = z

(normalize)

2
2
2
(b) f 2 ( z ) = z = ( x y ) + i 2 xy = u + iv

dx
= u = x2 y 2
dt
dy
2 xy
= 2
dx x y 2

dy
= v = 2 xy
dt

x 2 + y 2 = c3 y

f2 ( z ) = z 2

(normalize)

Circles that have centers at y-axis and


pass through the origin
26

Limits and Continuity


DEFINITION 17.4
Limit of a Function
Suppose the function f is defined in some neighborhood of z0, except
possibly at z0 itself. Then f is said to possess a limit at z0, written

lim f ( z ) = L

z z0

if, for each > 0, there exists a > 0 such that |f(z) L| < whenever
0 < |z z0| <
z-plane

f(z)-plane

Note
1. For a real function

x0

f ( x ) L as x x0

lim f ( x ) = L

x x0

Along the real axis


2. For a complex function
f ( z ) L as z z0

lim f ( z ) = L

z z0

along any direction

z0
x

THEOREM 17.1

27

Limit of Sum, Product, Quotient

f ( z ) = L1 and lim g ( z ) = L2, then


Suppose zlim
z
zz
0

lim f ( z ) + g ( z ) = L1 + L2

z z0

lim f ( z ) g ( z ) = L1 L2

z z0

lim

z z0

f ( z ) L1
= , L2 0
g ( z ) L2

DEFINITION 17.5
Continuity at a Point
f ( z ) = f ( z0 )
A function f is continuous at a point z0 if zlim
z
0

function f and g are continuous at z0, then

( f + g ) is continuous at z0
( fg ) is continuous at z0
f
g

is continuous at z0, g ( z0 ) 0

28

f ( z ) = a0 + a1 z + ...... + an z n =

n
k =0

ak z k, an 0 , n: non-negative integer

f(z): polynomial of degree n


lim z = z0 ,...... lim z n = z0n

z z0

f ( z) =

z z0

n
k =0

ak z k is continuous everywhere

g ( z ) , h ( z ): polynomial functions
g ( z)
is continuous except at points at which h(z) = 0
rational function f ( z ) =
h( z)

Derivative
z = x + iy

DEFINITION 17.6
Derivative
Suppose the complex function f is defined in a neighborhood of a point
z0. The derivative of f at z0 is
f ' ( z0 ) = lim

z 0

provided this limit exists.

f ( z0 + z ) f ( z0 )
z

29

f differentiable at z0

f ' ( z0 ) exists

Note:
dw
1. w = f ( z ), derivative f ' ( z ) =
dz

2. f is differentiable at z0

f is continuous at z0

Rules of differentiation
f (z) & g (z): differentiable at z; c: complex constant
Constant rules
d
d
c=0
cf ( z ) = cf ' ( z )
dz
dz
Sum rule
d
f ( z ) + g ( z ) = f '( z ) + g '( z )
dz
Product rule
d
f ( z ) g ( z ) = f ( z ) g '( z ) + f '( z ) g ( z )
dz

Quotient rule

g ( z ) f '( z ) f ( z ) g '( z )
d f ( z)
=
dz g ( z )
g2 ( z)
Chain rule
d
f g ( z ) = f ' g ( z ) g '( z )
dz
Differentiation of powers of z
d n
z = nz n 1 , n = integer
dz

( )

30

Using the Rules of Differentiation

Example 3

Differentiate (a) f ( z ) = 3z 4 5 z 3 + 2 z and (b) f ( z ) =

Sol:

z2
4z +1

(a) f ' ( z ) = 3 4 z 3 5 3z 2 + 2 = 12 z 3 15 z 2 + 2
(b) f ' ( z ) =

2 z ( 4 z + 1) z 2 4

( 4 z + 1)

Note: f ' ( z0 ) exists


lim

z 0

Example 4

4z2 + 2z

( 4 z + 1)

f is differentiable at z0

f ( z + z ) f ( z )
exists, same complex number from any direction
z

A Function that is Nowhere Differentiable

Show that the function f ( z ) = x + 4iy is nowhere differentiable


Sol:

z = x + iy
lim

z 0

f ( z + z ) f ( z ) = ( x + x ) + 4i ( y + y ) ( x + 4iy ) = x + 4iy

f ( z + z ) f ( z )
x + 4iy
= lim

0
z
x + iy

(i) along x axis y = 0, z = x 0


lim

z 0

lim

z 0

x = 0, z = iy 0
x + 4iy
4iy
lim
= lim
=4
z 0 x + iy
iy 0 iy

(ii) along y axis

x + 4iy
x
= lim
=1

0
x + iy
x
f ( z + z ) f ( z )
does not exist
z

x + 4iy differentiable at any z

31

Analytic Functions
DEFINITION 17.7
Analyticity at a Point
A complex function w = f (z) is said to be analytic at a point z0 if f is
differentiable at z0 and at every point in some neighborhood of z0
f is analytic in a domain D

f is analytic at every point in D

Note:
1. Analyticity at a point is a neighborhood property
2. Analyticity at a point is not the same as differentiability at a point

Example
f ( z) = z

- differentiable at z = 0
- differentiable nowhere else
- nowhere analytic

f ( z ) = z2

- differentiable at every point in a complex plane


- analytic everywhere

Later, an example show that f is differentiable along a line only


f is not analytic

32

f is entire function
f ( z) =

n
k =0

f is analytic everywhere

ak z k : polynomial function entire function

f ( z ) = z 4 + 5z 2 + 4

= ( z 2 + 1)( z 2 + 4 )

= ( z + i )( z i )( z + 2i )( z 2i )

zero of f : i, i, 2i, 2i

In algebra:
A number c is a zero of a polynomial function f (x)
Def. z0 is a zero of f (z)

(x c ) is a factor of f (x).

f (z0) = 0.
33

Homework 1
17.1
18, 30, 36, 38

17.2
16, 20, 30, 34

17.3
8, 16, 24, 26

17.4
12, 30, 34, 38, 40

34

Contents
17.1 Complex Numbers
17.2 Powers and Roots
17.3 Sets in the Complex Plane
17.4 Functions of a Complex Variable
17.5 Cauchy-Riemann Equations
17.6 Exponential and Logarithmic Functions
17.7 Trigonometric and Hyperbolic Functions
17.8 Inverse Trigonometric and Hyperbolic Functions

35

A Necessary Condition for Analyticity (Differentiability)


THEOREM 17.2

Cauchy-Riemann Equations

Suppose f ( z ) = u ( x, y ) + iv ( x, y ) is differentiable at a point z = x + iy . Then at z


the first order derivatives of u and v exist and satisfy the Cauchy-Riemann
equations
u v
u
v
=
and
=
x y
y
x

Proof:

f ' ( z ) = lim

also, f ' ( z ) =

f ( z + z ) f ( z )

z
f ( z ) = u ( x, y ) + iv ( x, y )
z 0

f u v = i f = i u + v
=
+i
y
y y
x x x

exists same value as z 0 along any direction

z = x + iy

f ( z + z ) f ( z ) u ( x + x, y + y ) + iv ( x + x, y + y ) u ( x, y ) iv ( x, y )
=
z
x + iy

z
=

y
=
0
(i) z 0 horizontally,
g ( z) =

u ( x + x, y ) + iv ( x + x, y ) u ( x, y ) iv ( x, y ) u v
=
+i
x 0
x
x x

fi ' ( z ) = lim

36

(ii) z 0 vertically, x = 0 , z = iy
u ( x, y + y ) + iv ( x, y + y ) u ( x, y ) iv ( x, y )
u v
= i +
y 0
iy
y y

fii ' ( z ) = lim

fi ' ( z ) = fii ' ( z )

f ' ( z ) exists

u v
u
v
=
and
=
x y
y
x

Necessary condition for analyticity


f ( z ) = u ( x, y ) + iv ( x, y ) is analytic throughout a domain D
u v
u
v
=
and
=
at every point in D
x y
y
x

Using the Cauchy-Riemann Equations

Example 1

f ( z ) = z 2 + z = x 2 y 2 + x + i ( 2 xy + y ) is analytic for all z


2

u ( x, y ) = x y + x

v ( x, y ) = 2 xy + y

u
v
= 2x +1 =
x
y

u
v
= 2 y =
y
x

f ( z ) = z = x2 + y 2
2

u = x2 + y 2
v=0
Ex:
f (z) = |z|2 satisfies
CRE at 0, but not at
other points
37

Example 2

Using the Cauchy-Riemann Equations

Show that the function f ( z ) = ( 2 x 2 + y ) + i ( y 2 x ) is not analytic at any point

Sol:

u ( x, y ) = 2 x 2 + y

v ( x, y ) = y 2 x

u
v
u
v
= 4x
=2 y
=1 =1
y
x
y
x
Note: f satisfies CRE on the line y = 2x
u
v
=
at any point
f ( z ) is differentiable on the line y = 2x
y
x
u v
For z on y = 2x,
for y = 2 x
=
x y
N ( z ) satisfying f ( z ) differentiable in N ( z )
u v
for y 2 x

x y

f (z) nowhere analytic

38

A Sufficient Condition for Analyticity


Criterion for Analyticity

THEOREM 17.3

Suppose the real-valued function u ( x, y ) and v ( x, y ) are continuous and have


continuous first-order partial derivatives in a domain D. If u and v satisfy the
Cauchy-Riemann equations at all points in D, then
1. the complex function f ( z ) = u ( x, y ) + iv ( x, y ) is analytic in D.

u v u u
( u + iv ) = + i = i
x
x x x y

v u v v
=
( u + iv ) = i = + i
y y y x
iy

2. f ' ( z ) =

Proof:

f ( z ) is analytic in D

To show f ' ( z ) exists in D

Let P: (x, y) be any fixed point in D


Choose a point Q:

( x + x, y + y ) such that the segment PQ is in D

From mean-value theorem

u
x
v
v ( x + x, y + y ) v ( x, y ) = x
x

u ( x + x, y + y ) u ( x, y ) = x

u
y
M1
v
+ y
y
M2
+ y

M1

M2

39

Set f ( z ) = u ( x, y ) + iv ( x, y )

f = f ( z + z ) f ( z )

z = x + iy

f = u ( x + x, y + y ) + iv ( x + x, y + y ) u ( x, y ) + iv ( x, y )
f = x

f = x

f = x

u
x
u
x
u
x

+ y
M1

f = z

u
x

v
x

y
M1

+ iy
M1

+iy

+ i x
M1

+ i x
M1

u
x

u
x

+ iy
M1

u
y

+ i x
M2

iy
M1

u
x

M2

u
x
u
x

v
x
v
x
v
x

+ y
M2

+ y
M2

+ iy
M2

v
y
u
x

M1

+ i z
M1

v
x

u
v
=
y
x
u v
=
x y

M2

v
x

+ix

CR eq.

M2

M1

v
x

ix
M1

+ x
M1

v
x

v
x

M1

M2

v
x

M1

40

f = z

u
x

+ iy
M1

u
x

M2

u
x

+ i z
M1

v
x

+ x
M1

v
x

M2

v
x

M1

Divide by z and let z 0

f
= lim
z 0 z
z 0
lim

u
x

u v
f '( z ) =
+i
x x

+i
M1

v
x

y
z

+ i lim

z 0

M1

u
v
x
x
At P: (x,y)
(continuous)

u
x

M2

u
x

+
M1

x
z

v
x

M2

v
x

M1

0
x
y
1,
1
z
z

u
x

f ' ( z ) exists at every point z in D

M2

u
x

0,
M1

v
x

M2

v
x

0
M1

f ( z ) is analytic in D

41

Example 3
f ( z ) = u + iv =

Using Theorem 17.3


x
y
i 2
2
x + y2
x +y
2

u
y 2 x2
v
=
=
2
2
2
x ( x + y )
y

(= 1/z)

u
2 xy
v
=
=
2
y ( x 2 + y 2 )
x

except z = 0

f ( z ) is analytic in D = { z z 0}

Example

f ( z ) = z 2 = ( x 2 y 2 ) + i 2 xy = u + iv
u
v
= 2x =
x
y

u
v
= 2 y =
y
x

( x, y )

f ( z ) is analytic everywhere
f '( z ) =

u u
( u + iv ) = i = 2 x + i 2 y = 2 z
x
x y
42

Note: Analyticity (at a point)

Thm: If the real valued functions u(x, y) and v(x, y)


C1 in a N(z), and u(x, y), v(x, y) CRE at z, then f (z) =

Example
f ( z ) = u + iv = x iy
2

u
v
=0=
y
x

u(x, y) + i v(x, y) is differentiable at z.


f ( z ) is differentiable on y = x (not a domain)

( x, y )

u v
=
x y

for y = x

u v

x y

for y x

Differentiability (at a point)

f ( z ) is nowhere analytic

For z on y = x,
N ( z ) satisfying f ( z ) differentiable in N ( z )

Harmonic Functions
DEFINITION 17.8
Harmonic Functions
A real-valued function ( x, y ) that has continuous second-order partial
derivatives in a domain D and satisfies Laplaces equation is said to be
harmonic in D
Laplaces equation: 2 ( x, y ) =

2
2
( x, y ) = 0
+
x 2 y 2
43

THEOREM 17.4

A Source of Harmonic Function

Suppose f ( z ) = u ( x, y ) + iv ( x, y ) is analytic in a domain D. Then the functions


u ( x, y ) and v ( x, y ) are harmonic functions.
Proof:

Assume u and v have continuous second-order partial derivatives


CR eq.

u u
v
=
=
2
x
x x
x y
2

u u
v
=
=
2
y
y y
y x
2

2u 2u
+
=0
x 2 y 2

u is a harmonic function

2v 2v
+
=0
x 2 y 2

v is a harmonic function

u
v
=
y
x
u v
=
x y

Conjugate Harmonic Function


u(x,y): a given harmonic function in domain D, v(x,y) is conjugate harmonic
function of u(x,y)
v(x,y) is a harmonic function in D so that
f ( z ) = u ( x, y ) + iv ( x, y ) is analytic in D
44

Example 4

Harmonic Function/Conjugate Harmonic Function

(a) Verify that the function u ( x, y ) = x3 3xy 2 5 y is harmonic in the entire


complex plane. (b) Find the conjugate harmonic function of u.

Sol:

(a)

u
= 3x 2 3 y 2
x
u
= 6 xy 5
y

2u
= 6x
x 2
2u
= 6 x
y 2

2u 2u
+
=0
x 2 y 2

(b) Let v(x,y) is conjugate harmonic function of u(x,y)


f ( z ) = u ( x, y ) + iv ( x, y ) is analytic CR eq.
u
v
v u
=
=
= 3x 2 3 y 2
y
x
y x
u v
v
u
=
=
= 6 xy + 5
x y
x
y
v
= 6 xy + h ' ( x ) = 6 xy + 5
v = 3x 2 y y 3 + h ( x )
x
h ( x ) = 5x + c

h '( x) = 5

v = 3x 2 y y 3 + 5 x + c

The analytic function: f ( z ) = x3 3xy 2 5 y + i ( 3x 2 y y 3 + 5 x + c )


(= z3 + i5z + c)

45

Problem 32
Suppose u and v are the harmonic functions forming the real and imaginary
parts of an analytic function. Show that the level curves u(x,y) = c1 and v(x,y) =
c2 are orthogonal.
Sol:
u =

u u
i+
j
x
y

u v =

v =

v v
i+ j
x y

Since the gradients of u and v are orthogonal vectors, the level


curves u(x,y) = c1 and v(x,y) = c2 are orthogonal.

u v u v v v v v
+
=

=0
x x y y y x x y

Note: u(x,y) and v(x,y) are harmonic functions satisfy


f(z) = u(x,y) + iv(x,y) analytic
u(x,y) = c1 and v(x,y) = c2 are orthogonal

Electrostatics
u(x,y) = c1 (equipotential curves)

v(x,y) = c2 (electric field lines)

46

Contents
17.1 Complex Numbers
17.2 Powers and Roots
17.3 Sets in the Complex Plane
17.4 Functions of a Complex Variable
17.5 Cauchy-Riemann Equations
17.6 Exponential and Logarithmic Functions
17.7 Trigonometric and Hyperbolic Functions
17.8 Inverse Trigonometric and Hyperbolic Functions

47

Exponential Function
Real exponential function f ( x ) = e x
f '( x) = f ( x)

f ( x1 + x2 ) = f ( x1 ) f ( x2 )

Complex exponential function f ( z ) = e z


For y = 0

ez = ex

f '( z ) = f ( z )

f ( z1 + z2 ) = f ( z1 ) f ( z2 )

e x1 + x2 = e x1 e x2

z = x + iy
e z1 + z2 = e z1 e z2

For real x
ex =

k =0

xk
x2
xk
= 1 + x + + ...... + + ......
2!
k!
k!

For real y
e =
iy

( iy )

k =0

= 1

k!

( iy )
= 1 + iy +
2!

( iy )
+
3!

( iy )
+
4!

+ ......

y2 y4 y6
y3 y5 y7
+ ...... + i y + ......
2! 4! 6!
3! 5! 7!

= cos y + i sin y

Euler Formula
48

DEFINITION 17.9
e =e
z

x + iy

Exponential Function

= e ( cos y + i sin y )
x

e z = e x ( cos y + i sin y ) = u ( x, y ) + iv ( x, y )
u ( x, y ) = e x cos y

v ( x, y ) = e x sin y

u
v
= e x cos y =
x
y

u
v
= e x sin y =
y
x

f ( z ) = e analytic for all z


entire function
z

( x, y )

Th. 17.3
u, v: continuous
continuous first-order partial derivatives
Satisfy CR eq.

( u + iv ) = e x ( cos y + i sin y ) = e x ( cos y + i sin y ) = f ( z )


x
x

or i ( u + iv ) ... Thm. 17.3


d z
z
y
e =e
dz

f '( z ) =

49

f ( z1 ) f ( z2 ) = e z1 e z2 = e x1 ( cos y1 + i sin y1 ) e x2 ( cos y2 + i sin y2 )


= e x1 e x2
= e x1 + x2
= e z1 + z2

( cos y1 cos y2 sin y1 sin y2 ) + i ( sin y1 cos y2 + cos y1 sin y2 )


cos ( y1 + y2 ) + i sin ( y1 + y2 )
= f ( z1 + z2 )

e z1 e z2 = e z1 + z2

e z1
= e z1 z2
e z2

Periodicity
f ( z ) = e z is periodic with complex period 2i

f ( z + 2n i ) = f ( z ) , n = 0, 1, 2,....
e 2 n i = cos 2n + i sin 2n = 1
e z + 2 n i = e z e 2 n i = e z z

50

Complex plane

horizontal strips Sn

Sn = z = x + iy ( 2n 1) < y ( 2n + 1) , n = 0, 1, 2,...

S1

z S0 , e z = f ( z ) = f ( z + 2n i )

S0

S0 = { z = x + iy < y }: fundamental region for ez

S1

The range of any Sn is the entire complex plane

Polar Form of a Complex Number


z = x + iy

: rectangular form of z

z = r ( cos + i sin ) : polar form of z


z = rei

: exponential form of z

Flow of ez
51

i (t )

Application to AC Circuits
L

di
1
idt = e ( t ) = E0 sin t
+ Ri +
dt
C

e ( t ) = E0 sin t = Im ( E0 e

jt

) = Im ( E )

i ( t ) = I 0 sin t = Im ( I 0 e jt ) = Im ( I )

E = E0 e

e (t )

jt

I = I 0 e jt

dI
1
I
1
Idt = j LI + RI +
+ RI +
= j L + R +
I =E
dt
C
jC
jC

dI d
= ( I 0 e jt ) = j ( I 0 e jt ) = j I
dt dt

Idt = I 0 e jt dt =

1
I
I 0e jt ) =
(
j
j

Complex impedance
E
1
Z = = R + j L +
= Z e j
I
jC

I=

1
Z = R2 + L
C

= tan 1

L
R

1
C

jt

E
E E0e
=
= 0 e j (t )
j
Z Ze
Z

i p ( t ) = Im { I } = Im

E0 j (t )
E
= 0 sin (t )
e
Z
Z

52

Logarithmic Function
Logarithmic of real number x (x > 0): y = ln x e y = x

( x > 0)

Logarithmic of complex number z (z 0):


w = ln z e w = z ( z 0)

ew 0

ln 0 : undef .

ln z = w = u + iv
z = rei = e w = eu +iv = eu eiv

r = eu

u = log e r = log e z

ei = eiv

v = = arg z
= 0 + 2n = Arg z + 2n , n = 0, 1, 2,......

DEFINITION 17.10
For z 0 ,

Logarithm of a Complex Number

ln z = log e z + i arg z

= log e z + i ( Arg z + 2n ) , n = 0, 1, 2,......


Ln z = log e z + iArg z: principal value of ln z

-valued
single-valued

Arg z = 0 , < 0

53

Example 2 & 4 Complex Values of the Logarithmic Function


Find the values of (a) ln (2), (b) ln i, and (c) ln (1 i).
Sol: ln z = log e z + i ( Arg z + 2n ) , n = 0, 1, 2,......
Ln z = log e z + iArg z

Arg ( 2 ) =

(a) log e 2 = log e 2 = 0.6932

ln ( 2 ) = 0.6932 + i ( + 2n ) , n = 0, 1, 2,......
Ln ( 2 ) = 0.6932 + i

(b) log e i = log e 1 = 0 Arg i =

(c)

ln ( i ) = i

Ln ( i ) = i

+ 2n , n = 0, 1, 2,......

2
log e 1 i = log e 2 = 0.3466

Arg ( 1 i ) =

3
+ 2n , n = 0, 1, 2,......
4
3
Ln ( 1 i ) = 0.3466 i
4

ln ( 1 i ) = 0.3466 + i

54

Solving an Exponential Equation

Example 3

Find all values of z such that e z = 3 + i


Sol: z = ln
log e

3 +i

3 + i = log e 2 = 0.6932

z = 0.6932 + i

Arg

3 + i = tan 1

=
6
3

n = 0, 1, 2,......

+ 2n

Branch
ln z = log e z + i arg z = log e z + i ( Arg z + 2n ) , n = 0, 1, 2,......

-valued

collection of logarithmic functions = branches


Each function in the collection = branch of ln z
Each branch = function single-valued
f1 ( z ) = Ln z = log e z + iArg z : principal branch of ln z

D1 = { z z 0, < Arg z }

principal logarithmic function

f 2 ( z ) = ln 2 z = log e z + i arg 2 z : second branch of ln z

D2 = { z z 0, < arg 2 z 3 }

55

Branch: A branch of a multi-valued function is a single-valued function analytic


in some domain. At every point of the domain, the single-valued function must
assume exactly one of the various possible values that the multi-valued function
can assume.
Branch cut: a line used to create the a domain of analyticity
Branch point: a shared common point of all the possible branch cuts
Cut planes: D1, D2
Riemann surface

Branch cut, or branch line: portion of a line or curve introduced to define a


branch F of a multi-valued function f ; points on branch cut of F are not analytic.
v

2nd
branch

C F
A D

w = ln z

E B

z=e

principal
branch

ln z = log e z + i arg z = log e z + i ( Arg z + 2n ) , n = 0, 1, 2,......

w2 E

C, D

w1 B

56

Riemann Surfaces
w = ln z

Dn

D2
D1

57

http://en.wikipedia.org/wiki/Riemann_surface

Consider the mapping defined by w = u + iv = z 2

or z = w 1/2

Every point w 0 corresponds to two points z (z1, z1)


Right half of the z-plane (including the positive y-axis)
y

w = u + iv = z 2

Image of right half


of the z-plane

Left half of the z-plane (including the negative y-axis)


y

w = u + iv = z 2
x

Image of left half of


the z-plane
u

58

Image of right half of the z-plane: upper sheet


Image of left half of the z-plane: lower sheet
A point passes from right half to the left half of the z-plane
Image point passes from the upper to the lower sheet
Join the two sheets crosswise along the cut, negative real axis
Riemann Surface

59

ln ( z1 z2 ) = ln z1 + ln z2

ln

z1
= ln z1 ln z2
z2

Ln ( z1 z 2 ) ?= Ln z1 + Ln z 2
Ln (

Properties of Logarithms

Example 5

z1 ?
) = Ln z1 Ln z 2
z2

Suppose z1 = 1 and z2 = 1 . Then if we take ln z1 = 2 i and ln z2 = i , we get


ln ( z1 z2 ) = ln ( 1) = ln z1 + ln z2 = 2 i + i = 3 i
ln

z1
= ln ( 1) = ln z1 ln z2 = 2 i i = i
z2

Analyticity
f ( z ) = Ln z = log e z + iArg z = u + iv

continuous at z = 0

f(0) = Ln 0: undef.

continuous on negative real axis

z
x0
z

- < Arg z < , x0 negative real axis


z x0 from upper half plane, Arg z
z x0 from lower half plane, Arg z
60

Principal branch of ln z
f ( z ) = Ln z analytic on non-positive real axis
analytic in cut plane D = { z z 0, < Arg z < }

Non-positive real axis: branch cut for Ln z


1) f ( z ) = Ln z analytic in D = { z z 0, < Arg z < }
2)

d
1
Ln z = , z D
dz
z

Complex Powers
For real variables, x a = ea log

w = Ln z as a flow

For complex variables, : complex number, z = x + iy ( 0)


z = e ln z , ( z 0) : complex power of z

multi-valued

z = e Ln z , ( z 0) : principal value of z

single-valued
61

= n ( n=0, 1, 2,......) , z = z n single-valued

Example
= 2 ; z = rei , = arg z

z = e ln z = e

2( log e r + i )

= e2loge r ei 2 = r 2 ei ei = ( rei )( rei ) = z 2

Complex Power

Example 6

Find the value of i 2i

Note:

z =1

z =i

Sol: = 2i

log e z = log e 1 = 0

ln z

i = z =e

=e

2i i

+ 2n

1+ 4 n

+ 2n = i

(i ) = i (i )
( z ) = z , for integer n
i 2

z = e ln z

arg z =

ln z = log e z + i arg z = 0 + i
2i

single-valued

1 + 4n

2 i

2i

but not for n complex

= e(1+ 4 n ) , n = 0, 1, 2,......

i 2i = e = 0.0043
n=0
62

Contents
17.1 Complex Numbers
17.2 Powers and Roots
17.3 Sets in the Complex Plane
17.4 Functions of a Complex Variable
17.5 Cauchy-Riemann Equations
17.6 Exponential and Logarithmic Functions
17.7 Trigonometric and Hyperbolic Functions
17.8 Inverse Trigonometric and Hyperbolic Functions

63

Definitions
Real variable x
Euler formula: e ix = cos x i sin x
eix e ix
2i
x
e e x
sinh x =
2

eix + e ix
2
x
e + e x
cosh x =
2

sin x =

cos x =

DEFINITION 17.11 Trigonometric Sine and Cosine


For any complex number z = x + iy,
sin z =

tan z =

sin z
cos z

eiz eiz
2i
cot z =

cos z =
1
tan z

eiz + eiz
2
sec z =

1
cos z

csc z =

1
sin z

64

DEFINITION 17.12 Hyperbolic Sine and Cosine


For any complex number z = x + iy,
sinh z =
tanh z =

sinh z
cosh z

e z e z
2
coth z =

cosh z =
1
tanh z

e z + e z
2

sech z =

1
cosh z

csch z =

1
sinh z

Analyticity
e iz entire functions
sin z = 0

z = n

cos z = 0

( 2n + 1)
z=

n: integer

1) sin z, cos z entire functions


2) tan z, sec z analytic except at z = (

2n + 1)
, n = 0, 1, 2,......
2
3) cot z , csc z analytic except at z = n , n = 0, 1, 2,......
65

4) sinh z , cosh z entire functions


5) tanh z , sech z analytic except at z =

( 2n + 1) i , n = 0, 1, 2,......
2

6) coth z , csch z analytic except at z = n i, n = 0, 1, 2,......

Derivatives
d z
e = ez
dz

d iz
e = ie iz
dz

d
d eiz eiz
ieiz + ieiz eiz + eiz
=
=
= cos z
sin z =
dz
dz
2i
2i
2

......

d
sin z = cos z
dz
d
tan z = sec2 z
dz
d
sec z = sec z tan z
dz
d
sinh z = cosh z
dz

d
cos z = sin z
dz
d
cot z = csc2 z
dz
d
csc z = csc z cot z
dz
d
cosh z = sinh z
dz
66

Identities

z
z
1) sin ( iz ) = i sinh z sin ( iz ) = 1 ei(iz ) ei(iz ) = 1 e z e z = i e e = i sinh z

2i
2i
2
1
1
cos ( iz ) = cosh z cos ( iz ) = ei(iz ) + ei(iz ) = e z + e z = cosh z
2
2
1
sinh ( iz ) = i sin z sinh ( iz ) = eiz eiz = i sin z
2
1
cosh ( iz ) = cos z cosh ( iz ) = eiz + eiz = cos z
2

2) sin( z ) = sin z

cos( z ) = cos z

rId25

sin( z1 z2 ) = sin z1 cos z2 cos z1 sin z2


cos( z1 z2 ) = cos z1 cos z2 sin z1 sin z2
sin 2 z = 2sin z cos z

cos 2 z = cos 2 z sin 2 z

sin( z + 2n ) = sin z , cos( z + 2n ) = cos z periodic functions of period 2


tan( z + n ) = tan z periodic functions of period
sinh( z + 2n i ) = sinh z , cosh( z + 2n i ) = cosh z

periodic functions of period 2i


67

3) sin z = sin x cosh y + i cos x sinh y


sin z = sin ( x + iy ) = sin x cos iy + cos x sin iy = sin x cosh y + i cos x sinh y
cos z = cos x cosh y i sin x sinh y

cos z = cos ( x + iy ) = cos x cos iy sin x sin iy = cos x cosh y i sin x sinh y
sinh z = sinh x cos y + i cosh x sin y

sinh z = i sin ( iz ) = i sin ( y + ix ) = i sin ( y ) cos ( ix ) + cos ( y ) sin ( ix )

= i [ sin y cosh x + i cos y sinh x ] = i sin y cosh x + cos y sinh x


cosh z = cosh x cos y + i sinh x sin y
cosh z = cos ( iz ) = cos ( y + ix ) = cos ( y ) cos ( ix ) sin ( y ) sin ( ix )

= cos y cosh x + i sin y sinh x


2

4) sin z = sin 2 x + sinh 2 y


sin z = ( sin z )( sin z ) = ( sin x cosh y + i cos x sinh y )( sin x cosh y i cos x sinh y )
2

= sin 2 x cosh 2 y + cos2 x sinh 2 y = sin 2 x + ( sin 2 x + cos 2 x ) sinh 2 y


= sin 2 x + sinh 2 y

1 + sinh 2 y

cos z = cos 2 x + sinh 2 y


68

Zeros
sin z = sin 2 x + sinh 2 y = 0, ( x, y )
2

sin z = 0

sin z = 0

sin 2 x = 0

x = n

sinh 2 y = 0

y=0

sin x cosh y + i cos x sinh y = 0

z = n , n = 0, 1, 2,......
cosh y = ( e y + e y ) / 2 0

x = n

sin x = 0

sin x cosh y = 0

sinh y = 0
y=0
cos x sinh y = 0
cos x 0, x = n
z = n , n = 0, 1, 2,......, zeros of sin z

sin z = 0
cos z = 0

z=

( 2n + 1) , n = 0, 1, 2,......, zeros of cos z


2

i sin ( iz ) = 0

sinh z = 0

z = n , n = 0, 1, 2,......

iz = m

z = m i = n i

sinh z = 0

z = n i, n = 0, 1, 2,...... , zeros of sinh z

cosh z = 0

z=

( 2n + 1) i , n = 0, 1, 2,...... , zeros of cosh z


2

69

Bounded and Unbounded


For real variable x,

sin x 1
cos x 1

For complex variable z,

sin z
cos z

, bounded
, unbounded

sin z = sin 2 x + sinh 2 y

unbounded

Solving a Trigonometric Equation

Example 2

Solve the equation: cos z = 10


Sol:

eiz + eiz
= 10
2

eiz + e iz = 20

e 2iz 20eiz + 1 = 0

eiz =

20 400 4
= 10 3 11
2

iz = ln 10 3 11 = log e 10 3 11 + i Arg 10 3 11 + 2n

=0

= log e 10 + 3 11 + i 2n
z = 2n

i log e 10 + 3 11

log e 10 3 11 = log e
= log e

(10 3 11)(10 + 3 11)


(10 + 3 11)
1
= log (10 + 3 11 )
(10 + 3 11)
e

70

Contents
17.1 Complex Numbers
17.2 Powers and Roots
17.3 Sets in the Complex Plane
17.4 Functions of a Complex Variable
17.5 Cauchy-Riemann Equations
17.6 Exponential and Logarithmic Functions
17.7 Trigonometric and Hyperbolic Functions
17.8 Inverse Trigonometric and Hyperbolic Functions

71

sin ( w + 2n ) = sin w
z = sin w

w + 2n

w = sin 1 z

w + 2n -valued

w
n=0

single-valued

principal value

Inverse Sine sin 1 z = arcsin z


w = sin 1 z
z = sin w =

z = sin w
e e
2i
iw

iw

e 2iw 2izeiw 1 = 0
eiw = iz 1 z 2 = iz + (1 z 2 )

(1 z )

2 1/ 2

1/ 2

iw = ln iz + (1 z 2 )

w = sin 1 z = i ln iz + (1 z 2 )

1/ 2

1/ 2

= 1 z2

-valued

sin 1 z = i ln iz + (1 z 2 )

1/ 2

cos 1 z = i ln z + i (1 z 2 )

1/ 2

i i+z
tan 1 z = ln
2 iz

72

Values of an Inverse Sine

Example 1

Find all values of sin 1 5


Sol: sin 1 z = i ln iz + 1 z 2 1/ 2
( )

(1 z

2 1/ 2

= 1

( )
5

2 1/ 2

z= 5

= ( 4 )

1/ 2

sin 1 5 = i ln i 5 2i = i ln
= i log e

sin 1 5 =

5 2 +i

= 2i

52

+ 2n

i log e

5 + 2 , n = 0, 1, 2,......

( )
5

2 1/2

= 2e

+ m

= 2i

+ 2n

5+2

1/2

+ 2n i log e

i log e

= 4e

i ( + 2 m )

52 i

)
)

log e

n=0
1

( 5)

2 1/2

5 2 = log e
= log e

= 2i

(
(

52

)(

5+2

5+2

1
5+2

= log e

5+2

)
73

Derivatives
w = sin 1 z
1=

z = sin w

d
d
d
z = sin w =
sin w
dz
dz
dw

dw
dw
= cos w
dz
dz

dw
1
1
1
=
=
=
1/ 2
1/ 2
2
dz cos w (1 sin w )
(1 z 2 )

Example 2

d
1
sin 1 z =
1/ 2
dz
(1 z 2 )

d
1
cos 1 z =
1/ 2
dz
(1 z 2 )
d
1
tan 1 z =
1+ z2
dz

Evaluating a Derivative

Find the derivative of w = sin 1 z at z = 5


Sol: z = 5

(1 z )

2 1/ 2

dw
dz

z= 5

= 1

( 5)

2 1/ 2

= ( 4 )

1/ 2

= 2i

1
i
=
2i
2

74

Inverse Hyperbolic Functions


sinh z = ln z + ( z + 1)
1

1/ 2

d
1
cosh 1 z =
1/ 2
dz
( z 2 1)

cosh 1 z = ln z + ( z 2 1)

1/ 2

1 1+ z
tanh 1 z = ln
2 1 z

Example 3

d
1
sinh 1 z =
1/ 2
2
dz
( z + 1)

d
1
tanh 1 z =
1 z2
dz

Values of an Inverse Hyperbolic Cosine

Find all values of cosh1 (1)


Sol:

cosh 1 ( 1) = ln ( 1) = log e 1 + Arg ( 1) + 2n i

0
cosh

( 1) = ( 2n + 1) i, n = 0, 1, 2,......

75

Homework 2
17.5
6, 20, 28, 29

17.6
28, 38, 42, 46, 48

17.7
12, 14, 22

17.8
6

76

Chapter 18
Integration in the Complex Plane

77

Contents
18.1 Contour Integrals
18.2 Cauchy-Goursat Theorem
18.3 Independence of Path
18.4 Cauchys Integral Formulas

78

Contour

B = z (b)

C = z z ( t ) = x ( t ) + iy ( t ) , a t b : curve in complex plane


t : curve parameter
t:a b

z (t )

z (t ) : z ( a ) = A z (b) = B

A = z ( a ) : initial point of C

A = z (a)

B = z ( b ) : terminal point of C

Example

C = z z ( t ) = cos t + i sin t = eit , 0 t 2

t=a

C oriented with positive direction

~ increasing values of t

t =b

x = t3 , y = t5 , 1 t 1
x ' ( t ) = y ' ( t ) = 0 at z = 0

unit circle centered at 0


z(t) = x(t) + iy(t): tangential vector at t
C = { z z ( t ) = x ( t ) + iy ( t ) , a t b} smooth curve
x ' ( t ) , y ' ( t ) continuous on [a, b]
0 simultaneously on (a, b)
79

C = z z ( t ) = x ( t ) + iy ( t ) , a t b piecewise smooth curve


C=

n
j =1

C j , each Cj smooth

C does not cross itself


{
}
A=B
C = { z z ( t ) = x ( t ) + iy ( t ) , a t b} closed curve
C simple and closed
C = { z z ( t ) = x ( t ) + iy ( t ) , a t b} simple closed curve
C = z z ( t ) = x ( t ) + iy ( t ) , a t b simple

Smooth curve

Piecewise smooth
curve

Simple closed
curve

Closed but
not simple

In complex variables, contour or path = piecewise smooth curve

Contour integrals
Integral of complex function f (z) on contour C,
: contour integral or complex line integral

f ( z ) dz or

f ( z ) dz

80

f (z) = u(x,y) + iv(x,y)

1. Given C = { z z ( t ) = x ( t ) + iy ( t ) , a t b} smooth curve,


f ( z ) = u ( x, y ) + iv ( x, y ) complex function defined at all point on C
2. C =

n
j =1

C j ; [ a, b ] : a = t0 < t1 < ...... < tn = b

zk = z ( tk ) , k = 0,1, 2,......, n
y

zk = zk zk 1 , k = 1, 2,......, n

3. P = norm of partition = zk
4. zk* = z ( tk* ) , tk* ( tk 1 , tk )
n

5.

k =1

f ( z )zk
*
k

max

z0 z *
*
1 z
1 z2
z2
t0 = a *
t1 t1 *
t2 t2

zk* zk
zk 1
tk
*
t
k
t

*
zn 1 zn z
n
*
tn 1 t t = b
n

k 1

81

DEFINITION 18.1

Contour Integral

Let f be defined at points of a smooth curve C defined by x = x(t), y = y(t), a t


b. The contour integral of f along C is
C

f ( z ) dz = lim

P 0

n
k =1

f ( zk* ) zk

1. C smooth or piecewise smooth


2. f ( z ) continuous on C
f ( z ) dz exists Existence theorem
C

A Method of Evaluation
C

f ( z ) dz = lim

f z = lim

( u + iv )( x + iy )

( ux vy ) + i ( vx + uy )
( udx vdy ) + i C ( vdx + udy )
C

= lim
C

f ( z ) dz =

Sum of real-line integrals

82

C : x = x ( t ) , y = y ( t ) , dx = x ' ( t ) dt , dy = y ' ( t ) dt , a t b

z ( t ) = x ( t ) + iy ( t ) , dz = x ' ( t ) + iy ' ( t ) dt = z ' ( t ) dt

{
= {u

f z ( t ) z ' ( t ) dt = u x ( t ) , y ( t ) + iv x ( t ) , y ( t )

x ' ( t ) + iy ' ( t ) dt

x ( t ) , y ( t ) x ' ( t ) v x ( t ) , y ( t ) y ' ( t ) dt

+ i v x ( t ) , y ( t ) x ' ( t ) + u x ( t ) , y ( t ) y ' ( t ) dt
C

f ( z ) dz =

f ( z ) dz =

( udx vdy ) + i C ( vdx + udy )


{u x ( t ) , y ( t ) x ' ( t ) v x ( t ) , y ( t )
a
C
b

+i
C

f ( z ) dz =

b
a

b
a

{v

y ' ( t ) dt

x ( t ) , y ( t ) x ' ( t ) + u x ( t ) , y ( t ) y ' ( t ) dt

f z ( t ) z ' ( t ) dt

Evaluation of a Contour Integral

THEOREM 18.1

If f is continuous on a smooth curve given by z(t) = x(t) + iy(t), a t b, then


C

f ( z ) dz =

b
a

f z ( t ) z ' ( t ) dt

Complex integral

Real integrals
83

Evaluating a Contour Integral

Example 1
Evaluate
Sol:

z dz , where C is given by x = 3t , y = t 2 , 1 t 4

C : z ( t ) = x ( t ) + iy ( t ) = 3t + it 2

z ' ( t ) = 3 + 2it

f z ( t ) = z ( t ) = ( 3t + it 2 ) = 3t it 2
C

zdz =
=

C
4
1

f ( z ) dz =

( 2t

b
a

f z ( t ) z ' ( t ) dt =

+ 9t )dt + i

4
1

4
1

( 3t it ) ( 3 + 2it ) dt
2

3t 2 dt = 195 + 65i

Evaluating a Contour Integral

Example 2

1
Evaluate
dz , where C is the circle x = cos t , y = sin t , 0 t 2
C z
Sol: C : z ( t ) = x ( t ) + iy ( t ) = cos t + i sin t = eit z ' ( t ) = ieit
1
Ex.
f z (t ) =
= e it
z (t )
b
2
1
dz = f z ( t ) z ' ( t ) dt =
( eit )( ieit ) dt
C z
a
0

=i

2
0

z dz

1/z2 dz

z dz

dt = 2 i

very important!!
Also, what if C is not unit circle?

84

To prove: use Thm 18.1

Properties

Properties of Contour Integrals

THEOREM 18.2

Suppose f and g are continuous in a domain D and C is smooth curve lying


entirely in D. Then
(i)
(ii)

k f ( z) d z = k

[ f ( z ) + g ( z )] d z =

f ( z) d z =

(iii)
C = C1 + C2

(iv)

f ( z ) d z , k =constant

f ( z ) dz =

Evaluate

C =C1 + C2

(x

f ( z) d z +

C2

g ( z) d z

C2

C1

f ( z) d z

f ( z ) dz

Evaluating a Contour Integral

Example 3
Sol:

C1

f ( z) d z +

(x

+ iy 2 ) dz, where C is the contour shown in the figure

+ iy 2 ) d z =

C1

(x

+ iy 2 ) d z +

C2

(x

+ iy 2 ) d z

Thm. 18.2
85

On C1: y = x (x: curve parameter)

z '( x) = 1+ i

z = x + iy = x + ix
C1

f ( z) d z =

1
0

f ( z ) z ' ( x )dx =

= (1 + i )

0 x 1

1
0

(x

+ ix 2 ) (1 + i )dx

2
x 2 dx = i
0
3
1

On C2: x = 1 (y: curve parameter)


z = 1 + iy
C2

f ( z) d z =

f ( z) d z =
C = C1 + C2

z '( y ) = i

C1

2
1

1 y 2

f ( z ) z ' ( y )dy =

f ( z) d z +

C2

2
1

(1 + iy ) idy = 73 + i
2

7 5
f ( z) d z = + i
3 3

Curve C = { z z ( t ) = x ( t ) + iy ( t ) , a t b}
diff. length: ds =

( dx ) + ( dy )
2

length of C: L = ds =

(or arc length)

b
a

x '(t )

x '(t )

ds
2

+ y ' ( t ) dt = z ' ( t ) dt = dz
2

+ y ' ( t ) dt =
2

b
a

z ' ( t ) dt =

integration sign to modify as

b
a

dy

dx

dz

86

A Bounding Theorem ML-inequality

THEOREM 18.3

If f is continuous on a smooth curve C and if f ( z ) M for all z on C, then


f ( z ) dz ML , where L is the length of C
C

Proof:

n
k =1

f ( zk* ) zk

n
k =1

f ( zk* ) zk =

n
k =1

f ( zk* ) zk M

zk M

k =1

z1 + z2 + ...... + zn z1 + z2 + ...... + zn
C

f ( z ) dz = lim

P 0

n
k =1

Lk = ML

Lk

f ( zk* ) zk ML

zk 1 zk

Find an upper bound for the absolute value of


|z| = 4
C : z = 4, L = 2 4 = 8

Sol:

k =1

zk Lk

A Bound for Contour Integral

Example 4

zk

ez
d z, where C is the circle
C z +1

e z = e x +iy = e x eiy = e x eiy = e x = e x e4


ez

x
z

e
e
e
=

= =M
z +1 z + 1 z 1 3

e
8 e
d z ML =
C z +1
3

z1 + z2 z1 z2

87

Circulation and Net Flux

C = z z ( t ) = x ( t ) + iy ( t ) , a t b simple closed curve

T: unit tangent vector to the positively oriented C


N: unit normal vector to the positively oriented C
T=

dx + idy

N=

dy idx

( dx ) + ( dy )
( dx ) + ( dy )
Tds = dx + idy = ( dx, dy )
Nds = dy idx = ( dy, dx )
f ( z ) = u ( x, y ) + iv ( x, y ) = ( u, v )
f Tds = ( u, v ) ( dx, dy ) = udx + vdy
f Nds = ( u, v ) ( dy, dx ) = udy vdx
C

f Td s =
f Nd s =

ds =

ud x + vdy = circulation of f around C

udy vdx = Net flux of f across C

dx dy

( dx ) + ( dy )
2

dx
dy

f ( z)
N

~ the flow f to rotate around C


~ sources or sinks of f inside the curve C
88

) (

f Tds + i

f Nds =

( udx + vdy ) + i C ( udy vdx )

( u iv )( dx + idy ) =
Circulation of f = C f Tds = Re C f ( z ) dz
=

Net flux of f =

f Nds = Im

f ( z ) dz

f ( z ) dz

Net Flux

Example 5

Given the flow f (z) = (1 + i)z, compute the circulation around, and the net flux
across the circle C: |z| = 1
Sol:

Circle C: z = eit , dz = ieit , 0 t 2


f ( z ) = (1 i ) z = (1 i ) e it
C

f ( z ) dz =

2
0

(1 i ) eit

= (1 + i )

2
0

Circulation of f = Re
Net flux of f = Im

( ie dt )
it

dt = 2 (1 + i )
C

f ( z ) dz = 2

f ( z ) dz = 2
89

Contents
18.1 Contour Integrals
18.2 Cauchy-Goursat Theorem
18.3 Independence of Path
18.4 Cauchys Integral Formulas

90

Simply and Multiply Connected Domains


C simple closed contour with positive (counterclockwise) orientation
C

domain D simply connected


Every simple closed contour C lying in D can be shrunk
to a point without leaving D. (encloses only points of D)
D has no holes in it
domain D multiply connected
D simply connected
D has holes in it
domain D doubly connected

D has one hole in it

domain D triply connected

D has two holes in it

91

Greens Theorem
1. P ( x, y ) , Q ( x, y ) have continuous first-order partial derivatives in
simply connected domain D
2. C simple closed contour in D

R = the region enclosed by C


C

P ( x, y ) dx + Q ( x, y ) dy =
R

Q P

dxdy
x y

Cauchys Theorem
1825, Louis-Augustin Cauchy (1789 1857, Paris, France)
Most important theorem in complex analysis

Th.: Cauchy Theorem


1. f analytic in simply connected domain D
f ' continuous in D
2. C any simple closed contour in D
C

f ( z )dz = 0

92

Proof: f ( z ) = u ( x, y ) + iv ( x, y ) analytic
f ' continuous

u v u
v
= ,
=
x y y
x

u, v have continuous first-order partial derivatives

Greens theorem
C

f ( z ) dz =
=

( u + iv )( dx + idy )
C
C

( udx vdy ) + i C ( vdx + udy )

P ( x, y ) dx + Q ( x, y ) dy

Q P

dxdy
x y

v u
u v

dxdy + i

dxdy = 0
x y
x y
R

1883, Edouard Goursat (France)

THEOREM 18.4

Cauchy-Goursat Theorem (CGT)

Suppose a function f is analytic in a simply connected domain D. Then for


every closed contour C in D,
f ( z )dz = 0
C

Without assumption of continuity of f

Can not apply Greens thm here

f ( z ) analytic at all points within and on a simple closed contour C,


C

f ( z )dz = 0

93

Applying the Cauchy-Goursat Theorem

Example 1
Evaluate

e z dz,where C is the curve shown in the figure

Sol: e z analytic everywhere


C: simple closed contour
C

e z dz = 0

Example 2

Applying the Cauchy-Goursat Theorem

dz
( y 5) = 1
2
Evaluate C 2 ,where C is the ellipse ( x 2 ) +
z
4

Sol:

f ( z) =

1
analytic everywhere except at z = 0
z2

dz
=0
C z2

C
5

analytic on and within C

94

Applying the Cauchy-Goursat Theorem

Example 3

Given the flow f ( z ) = cos z, compute the circulation around and net flux across
C, where C is the square with vertices z = 1, z = i, z = 1, and z = i.
Sol:
C

f ( z )dz =

cos zdz = 0

cos z is analytic everywhere


Circulation around C = Re C f ( z )dz = 0
Net flux across C = Im

f ( z )dz = 0

Deformation of Closed Contours


f analytic in doubly connected domain D

C , C1 simple closed contour satisfied C1 surrounds the hole and is interior to C


D

C1

95

Cut AB, K = C + AB + ( C1 ) + BA

D
A

f analytic on and within K


K

f ( z )dz =

Cauchy-Goursat Th.
C

AB

C1

BA

) f ( z ) dz = 0

C
C1

cancelled

f ( z ) dz =

C1

f ( z ) dz =

C1

f ( z ) dz

Principle of deformation of closed contour


C1: continuous deformation of C
Complicated C

Sol:

deform
C

dz
z i

Simpler C1

Applying Deformation of Contours

Example 4
Evaluate

deform

dz
, where C is the outer contour shown in the fig.
z i
C1: z ( t ) = i + eit dz = ieit dt
0 t 2

18.4 Cauchy Integral Formula (CIF)


96

Th.
C is any simple closed contour enclosing z0
2 i, n = 1
dz
C

( z z0 )

Proof:

z0

n 1

0,

C1

dz = ieit dt
dz
C

( z z0 )

dz

C1

( z z0 )

2
0

(e )

it n

( ie dt ) = i

it

ei (1 n ) t dt =

2 i, n = 1
n 1

0,

Cauchy Integral Formula for Derivatives (CIFD)

Example 5
Evaluate

5z + 7
dz , where C is the circle |z 2| = 2
z + 2z 3

Sol:

C
z =1 z = 2

z = 3

5z + 7
dz
z + 2z 3
2

= 2i

=0
Cauchy-Goursat Theorem
97

Cauchy-Goursat Theorem for Multiply Connected Domains


f is analytic in multiply connected domain D
(n holes)

C1

Ck

Cn

C, C1, C2, , Cn: simple closed contours


C encloses all Ck (k = 1, 2, , n)
Each Ck encloses each hole
f is analytic on C and Ck and at every point interior to C, but exterior to Ck
(k = 1, 2, , n)
Cuts, K = C + cuts + ( C1 ) + ( C2 ) + ... + ( Ck ) + ... + ( Cn )
f is analytic on and within K
K

Cauchy-Goursat Th.
C

f ( z ) dz =

C1

+... +

f ( z )dz =

cuts

C1

+... +

Ck

+... +

Cn

) f ( z ) dz = 0

0
Ck

+... +

Cn

f ( z ) dz
98

THEOREM 18.5 Cauchy-Goursat Theorem for Multiply Connected Domain


f analytic in multiply connected domain D (with n holes)
C, C1, , Cn simple closed contours with positive orientation st. C
enclosed all Ck (k = 1, 2, , n) and each Ck encloses each hole.
C

f ( z )dz =

Example 6

n
k =1

Ck

f ( z ) dz

Applying Theorem 18.5

Evaluate

1
dz , where C is the circle |z| = 3
z +1
2

Sol:
analytic at

1
dz
z +1
2

99

1
dz
z +1
2

Th. 18.5

Exercise
1. f is analytic in simply connected domain D
2. C is a any closed contour in D (but not simple)
C

f ( z )dz = 0 Why?
100

Homework 3
18.1
14, 20, 24, 31

18.2
8, 18, 21, 24

101

Contents
18.1 Contour Integrals
18.2 Cauchy-Goursat Theorem
18.3 Independence of Path
18.4 Cauchys Integral Formulas

102

D is a simply connected domain


z0, z1 are two points in D
C, C1 are two contours in D with initial point z0 and terminal point z1
: closed contour
f is analytic in D
K

f ( z )dz =

f ( z ) dz =

Cauchy-Goursat Th.

f ( z ) dz +

C1

C1

f ( z ) dz

f ( z ) dz =

f ( z ) dz

C1

f ( z ) dz = 0

Def. of Independence of Path

THEOREM 18.6

Analyticity Implies Path Independence

If f is an analytic function in a simply connected domain D, then


independent of the path C

: independent of path in D
dependent only on end points z0, z1

z1

C2
C

z0

C1

f ( z ) dz =

C2

f ( z ) dz

: deformation of contours

C1

Example 1

f ( z ) dz =

is

103

Choosing a Different Path

Evaluate
, where C is the contour with initial point
z = 1 and terminal point z = 1 + i shown in the figure
Sol:

: analytic everywhere
C1: x = 1, 0 y 1
z = x + iy = 1 + iy
dz = idy
C

2 zdz =

C1

2 zdz =

DEFINITION 18.3

1+ i
1

2 zdz = 2 ( 1 + iy ) idy = 1 2i
0

Antiderivative

Suppose f is continuous in a domain D. If there exists a function F such that


F(z) = f (z) for each z in D, then F is called an antiderivative of f.
F is antiderivative or indefinite integral of f :
f ( z ) dz = F ( z ) + C , C : complex constant
F ' ( z ) = f ( z ) , for all z in D

[Ex] f ( z ) = sin z
F ( z ) = cos z

F ' ( z ) = sin z = f ( z )

F is analytic in D
104

THEOREM 18.7

Fundamental Theorem for Contour Integrals

If f is continuous in a domain D and F is an antiderivative of f in D, then for


any contour C in D with initial point z0 and terminal point z1,
C

f ( z ) dz = F ( z1 ) F ( z0 )

Independence of Path

Proof: C smooth curve: z = z ( t ) , a t b


C

f ( z ) dz =

b
a

= F z (t )

Example 2
Sol:

t =b
t =a

F ' z ( t ) z ' ( t )dt =

b
a

d
F z (t )
dt

= F z ( b ) F z ( a ) = F ( z1 ) F ( z0 )

dt

chain rule

Using an Antiderivative

f ( z ) = 2z

Example 3

f z ( t ) z ' ( t )dt =

F ( z ) = z2

Using an Antiderivative

Evaluate
point z = 2 + i

,where C is any contour with initial point z = 0 and terminal

Sol:
105

Note:
1. If C closed contour

z1 = z0

f ( z ) dz = F ( z )

z0
z0

=0

2. Continuous function f has an antiderivative F in D


f ( z ) dz = F ( z1 ) F ( z0 ) : independent of path in D
C

Existence of antiderivative
Th.
1. f continuous in D
2. f ( z ) dz : independent of path in D
C

f has an antiderivative in D ( i.e., F ( z ) in D st. F ' ( z ) = f ( z ) , z D )


Proof: Let z0: fixed point in D
z1: any point in D
2

Define: F ( z )

z
z0

F ( z + z ) F ( z ) =

f ( s )ds
z +z
z0

To prove: F ' ( z ) = f ( z ) , z D

f ( s )ds

z
z0

f ( s )ds =

z +z
z

f ( s )ds

106

z1

Let z1: temporarily fixed point in D st. z1 + z D

F ( z1 + z ) F ( z1 )
1 z1 +z
f ( s )ds f ( z1 )
f ( z1 ) =
z
z z1
f ( z1 ) : constant
1 z1 +z
f ( s ) f ( z1 ) ds
=
z z1

z1 +z

f continuous at z1

> 0, > 0, st. f ( s ) f ( z1 ) < , whenever s z1 <

Choose z st. z <

z1

F ( z1 + z ) F ( z1 )
1
f ( z1 ) =
z
z

<

1
z

z1 +z
z1
z1 +z
z1

f ( s ) f ( z1 ) ds
f ( s ) f ( z1 ) ds

F ( z1 + z ) F ( z1 )
f ( z1 ) = 0
z

1
z =
z

f ( z1 )

F ( z + z ) F ( z )
= f ( z), z D
z 0
z

F ' ( z ) = lim
z1 = z D

107

Note:
If f analytic in simply connected domain D
1) f continuous in D
2)

f ( z ) dz independent of path in D

THEOREM 18.8

f has an antiderivative in D

Existence of an Antiderivative

If f is analytic in a simply connected domain D, then f has an antiderivative in D;


that is, there exists a function F such that F(z) = f(z) for all z in D

Example 4
Sol:

Using the Logarithmic Function

1
Evaluate
dz, where C is the contour shown in the figure
C1 z
D1 = { z x = Re z > 0, y = Im z > 0}
1
f ( z ) = analytic in D1 : simply connected domain
z
1
F ( z ) = Ln z analytic in D1 st. = ( Ln z ) ' in D1
z
2i 1
1

2i
dz =
dz = Ln z 3 = Ln 2i Ln 3 = log e 2 + i
log e 3
C1 z
3 z
2
= 0.4055 + 1.5708i

C1

D1

108

D2 = { z = x + iy z 0} simply connected
1
f ( z ) = analytic in D2 : doubly connected domain
z
1
( Ln z ) '
z
F ( z ) = Ln z analytic in D2

analytic in D3 = { z z 0, < Arg z < }

C2

1
dz Ln z
z
= 2 i

z0
z0

D2

x
C2

D3
x

Integration by parts
1. f, g analytic in simply connected domain D
2. C contour in D with initial point z0 and terminal point z1
z1
z0
z1
z0
z1
z0

f ( z )g ' ( z ) dz = f ( z ) g ( z )

d
f ( z ) g ( z ) dz =
dz

z1
z0

z1
z0

z1
z0

f ' ( z )g ( z ) dz

f ' ( z ) g ( z ) + f ( z ) g ' ( z )dz

f ( z ) g ' ( z )dz = f ( z ) g ( z )

z1
z0

z1
z0

f ' ( z ) g ( z )dz

109

Contents
18.1 Contour Integrals
18.2 Cauchy-Goursat Theorem
18.3 Independence of Path
18.4 Cauchys Integral Formulas

110

THEOREM 18.9

Cauchys Integral Formula

Let f be analytic in a simply connected domain D, and let C be a simple closed


contour lying entirely within D. If z0 is any point within C, then
f ( z0 ) =

Proof:

1
2 i

f ( z)
dz
z z0

Let C1 = { z | z z0 = r}: a circle within C

z0

analytic in D except at z0
C

deform

C1

f ( z)
dz =
C zz
0

f ( z)
dz
C1 z z
0
f ( z0 ) + f ( z ) f ( z 0 )
=
dz
C1
z z0
f ( z ) f ( z0 )
1
= f ( z0 )
dz +
dz
C1 z z
C1
z z0
0
2 i

z0

C1

111

f analytic at z0

f continuous at z0

any arbitrary small


Choose C1:

Th. 18.3 ML inequality


f ( z ) continuous on a

For z C1

smooth curve C
f ( z ) M z on C
L : length of C

f ( z)
dz = 2 if ( z0 ) +
z z0

f ( z0 ) =

1
2 i

C1

f ( z ) f ( z0 )
dz = 2 if ( z0 )
z z0

f ( z)
dz
z z0

More practical restatement of Th. 18.9


analytic at all points within and on a simple closed contour C
any point interior to C
f ( z)
multiply both sides by 2i:
1
f ( z0 ) =
dz
2 i C z z0
powerful tool for evaluating integrals

112

Example 1

Using Cauchys Integral Formula

Evaluate

, where C is the circle |z| = 2


C

Sol:

: analytic at all points on and within C

: a point within C

Example 2
Evaluate

Using Cauchys Integral Formula


, where C is the circle |z2i| = 4

Sol:

: the only point within C

C
z = 3i

: analytic at all points on and within C

z = 3i

113

Example 3

Flux and Cauchys Integral Formula

The complex function,


, where k = a + ib and z1 are complex
numbers, gives rise to a flow in the domain
. If C is a simple closed
contour containing z = z1 in its interior, then from the Cauchy integral formula
we have

Circulation around C =
Net flux across C =
1. If
2.

exterior of C

Both the circulation and net flux = 0


Circulation around C = 0

k > 0, z1: source of the flow

k < 0, z1: sink of the flow

114

Derivatives of Analytic Functions


THEOREM 18.10

Cauchys Integral Formula for Derivatives

Let f be analytic in a simply connected domain D, and let C be a simple closed


contour lying entirely within D. If z0 is any point within C, then
f(

n)

( z0 ) =

n!
2 i

f ( z)

( z z0 )

n +1

dz

Proof: 1) Prove for n = 1


D

f ( z0 ) =

1
2 i

( z z0 )

f ( z)
dz
z z0

z0

( z z0 )

115

=0

f ( z ) analytic

f ( z ) M z on C

f ( z ) bounded

L = length of C

or

Choose z

z z0

( z z0 )

f ( z)

z z0

1
z z 0 z

2
1

z0

, then z z0 z z z0 z
2
2

f ( z)

For z C , z
z

/ 2 z + z
0

: the shortest distance between points on C (z) and the point z0


z z0

z z0 z

dz z

L = z

2 ML

ML inequality

( n +1)
n
( z0 )
2) Assume f ( ) ( z0 ) to prove f

116

Note:
1. Analytic function possesses derivatives of all order
n
f ', f '',..., f ( ) ,... analytic at z0
If f analytic at z0
f ( n ) ( z0 ) =

n!
2 i

f ( z)

( z z0 )

n +1

dz

f ( z)
1
dz (Cauchys integral formula)
C
2 i
zs
f ( z)
1
1
dn
dn 1
dn
n
f ( ) (s) = n f (s) = n
dz =
f ( z)
n
C
C
ds
ds 2 i
zs
ds z s
2 i

2. f ( s ) =

f ( z)

n!
2 i

dz (Cauchys integral formula for derivatives)

( z s)
3. If f ( z ) = u ( x, y ) + iv ( x, y ) analytic

( u + iv )
1) f ' ( z ) = ( u + iv ) =
n +1

iy
x

2) u ( x, y ) , v ( x, y ) C

4.

f ( z)

( z z0 )

n +1

dz =

dz

f '' ( z ) =

2
2
u
iv
+
=
(
)
( u + iv )
iyx
x 2

2 i ( n )
f ( z0 ) , evaluation of integrals
n!
117

Example 4

Using Cauchys Integral Formula for Derivatives

z +1
dz, where C is the circle |z| = 1
C z + 4z3
Sol:
z +1
f ( z)
z +1
z +1
3
= z +3 4 =
z0 = 0, n = 2, f ( z ) =
, f '' ( z0 ) = f '' ( 0 ) =
n +1
4
3
z + 4z
z
z+4
32
( z z0 )
f ( z)
2 i ( n )
2 i
3
z +1
dz =
f ( z0 ) =
f '' ( 0 ) = i
dz =
n +1
4
3
C
C z + 4z
2!
32
n!
( z z0 )

Evaluate

Example 5

Using Cauchys Integral Formula for Derivatives


z3 + 3

Evaluate
C

Sol:

z3 + 3
C

z ( z i)

z ( z i)

dz =
2

dz, where C is the contour shown in the figure

z3 + 3
C1

z ( z i)
z3 + 3

dz +
2

z3 + 3
C2

z ( z i)

dz

z3 + 3
( z i ) dz +
z dz = 6 i + 2 2 + 3i = 4 1 + 3i
=
(
)
(
)
2
C1
C2
z
( z i)
Cauchys Integral Formula Cauchys Integral Formula for derivative
2

118

Liouvilles Theorem

L = length of C = 2 r

C = z z z0 = r : circle

ML-inequality and Cauchys integral formula for derivatives


f ( z) M , z C

f(

n)

( z0 ) =

n!
2 i

f ( z)

( z z0 )

n +1

dz =

n!
2

Cauchys inequality: f ( n ) ( z0 )

THEOREM 18.11

f ( z)

( z z0 )

n!M
rn

n +1

n! M
2 r n +1

dz
f ( z)

( z z0 )

n +1

( 2 r ) =

n!M
rn

M
r n +1

Liouvilles Theorem

The only bounded entire function are constants.


Proof: For any point z0

f bounded entire function


f analytic everywhere

M
0
r r
f ' ( z0 ) = 0 for all z0
f ' ( z0 )

f bounded

f ( z ) = constant

f = constant

f ( z ) M for all z

119

Fundamental Theorem of Algebra


Th. The equation of a non-constant polynomial being zero has at least one root.
P ( z ) = a0 + a1 z + ... + an z n ,

( a1 , a2 ,..., an ) are not all zero

= nonconstant polynomial
P ( z ) = 0 has at least one root

Proof: By contradiction
If P ( z ) 0 for all z
f ( z) =

1
a0 + a1 z + ... + an z n

f ( z) =

1
entire function
P( z)

f ( z ) bounded entire function

f ( z ) bounded for all z

f ( z) =

1
= constant
P( z)

contradiction to
P ( z ) = nonconstant polynomial

at least one complex number z st. P ( z ) = 0


120

Homework 4
18.3
2, 18, 20, 24

18.4
16, 20, 24

121

Chapter 19
Series and Residues

122

Contents
19.1 Sequences and Series
19.2 Taylor Series
19.3 Laurent Series
19.4 Zeros and Poles
19.5 Residues and Residue Theorem
19.6 Evaluation of Real Integrals
19.7 Summation of Series by the Residue Theorem
19.8 Inverse of Laplace Transforms

123

Sequences
Complex sequence { zn } function : n = positive integer
[Ex.]

zn = complex number

{ zn } = {1 + i n }

n= 1 , 2 , 3

, 4 , 5

zn = 1 + i , 0 , 1 i , 2 , 1 + i ,
Def.: { zn } converges to a complex number L
lim zn = L
n

> 0, positive integer N , st. zn L < , n > N

Def.: { zn } diverges
[Ex.]

{ zn } not convergent

{ zn } = {1 + i n } diverges

lim (1 + i n ) does not exist


n

124

Example 1

A Convergent Sequence

i n +1
converges, since
n
n:
1, 2, 3, 4, 5,...,
i 1
i 1
zn : 1,
,
,
,
,..., 0
2 3
4 5

The sequence

Criterion for Convergence

THEOREM 19.1

A sequence { zn } converges to a complex number L if and only if Re ( zn ) converges


to Re ( L ) and Im ( zn ) converges to Im ( L )
zn = xn + iyn

L = a + ib

xn converges to a, and yn converges to b

zn converges to L

Convergence of complex sequence

Convergence of real sequences

125

Example 2
The sequence
zn =

ni
converges to i.
n + 2i

ni
2n
n2
= 2
+i 2
= xn + iyn converges to 0 + i
n + 2i n + 4 n + 4

2n
converges to 0
n +4
n2
converges to 1
yn = 2
n +4

xn =

Series
Complex sequence { zn }
Complex series:

k =1

zk = z1 + z2 + z3 + ... + zn + ...

nth partial sum: Sn =

n
k =1

zk = z1 + z2 + z3 + ... + zn

{Sn }: sequence of partial sum of series

k =1

zk

126

{Sn } converges (to L)

zk converges

k =1

L = lim Sn = lim
n

k =1

zk =

zk = sum of series

k =1

zk diverges

k =1

az k 1 = a + az + az 2 + ... + az n 1 + ...

k =1
n

Sn =

az k 1 = a + az + az 2 + ... + az n 1

k =1

az + az 2 + ... + az n 1 + az n

zS n

S n zSn = a (1 z n )

Sn =

k =1

zk = z1 + z2 + z3 + ... + zn

1
= 1 + z + z 2 + z 3 + ...
1 z
z <1
1
2
3
= 1 z + z z + ...
1+ z
1 zn
1 + z + z 2 + z 3 + ... + z n 1 =
1 z
z 1
1
zn
n 1
2
= 1 + z + z + ... + z +
1 z
1 z

Geometric Series

Note:

not convergent

zk

k =1

Sn =

a (1 z n ) z < 1, n

1 z
a

=
,
z <1
k 1
2
az = a + az + az + ... 1 z
k =1
divergent, z 1

a
1 z

127

Example 3

The series

(1 + 2i )

5k

k =1

1 + 2i
=
5

If

k =1

(1 + 2i )

5k 1

k =1

Necessary Condition for Convergence


n

k =1

k =1

zk diverges

k + 5i
diverges
k

n + 5i
zn =
n

Note: The converse is not true.

The nth Term Test for Divergence

If lim zn 0 , then series

k 1

zk converges, then lim zn = 0

THEOREM 19.3

[Ex.]

1 + 2i
5
=
<1
5
5
1 + 2i
a
i
5
=
=
=
1 z 1 1 + 2i 2
5
z =

k =1

THEOREM 19.2

az k 1

z k 1 diverges, z 1

k =1

1 0

zn = z

n 1

z =1
n
z >1
n

128

DEFINITION 19.1
An infinite series

Absolute Convergence

k =1

Example 4

zk converges

( i / k ) is absolutely convergent since i / k = 1/ k and the real


(1/ k ) converges. Recall from calculus that a real series of the form
k

(1/ k ) is called a p-series and converges for


p

k =1

k =1

k =1

k =1

Absolute Converges

The series
series

zk is said to be absolutely convergent if

p > 1 and diverges for p 1 .

Absolute convergence implies convergence

ik
1
i
= i 2 2 + ... converges, since
2
2 3
k =1 k

|ik/k2|

1/k2 converges

k =1

k =1

129

THEOREM 19.4

Suppose

k =1

Ratio Test

zk is a series of nonzero complex term such that lim

zn +1
=L
zn

(i) If L < 1, then the series converges absolutely.


(ii) If L > 1 or L = , then the series diverges.
(iii) If L = 1, the test is inconclusive.

THEOREM 19.5

Suppose

k =1

Root Test

n z
zk is a series of complex term such that lim
n = L
n

(i) If L < 1, then the series converges absolutely.


(ii) If L > 1 or L = , then the series diverges.
(iii) If L = 1, the test is inconclusive.

Power Series

z0 ; a0 , a1 , ..., ak , ...: given complex numbers

k =0

ak ( z z0 ) = a0 + a1 ( z z0 ) + a2 ( z z0 ) + ... + ak ( z z0 ) + ...
k

: power series in (z z0), with center at z0


130

Circle of Convergence
Every complex power series has radius of convergence R
Circle of convergence: |z z0| = R
The power series
- Converges absolutely for |z z0| < R
- Diverges for |z z0| > R
Note:
R = 0,

k =0

R = finite,

ak ( z z0 ) converges only at z0
k

k =0

R = ,

k =0

k
ak ( z z0 ) converges at all interior points of the circle z z0 < R

ak ( z z0 ) converges for all z


k

In Theorem 19.4 and 19.5

k =1

zk

ratio test: lim


n

zn +1
zn

or

n z
root test: lim
n
n

131

For a power series

k =0

ak ( z z0 ) = a0 + a1 ( z z0 ) + a2 ( z z0 ) + ... + ak ( z z0 ) + ...,
k

z
lim n +1 and lim n zn depend only on the coefficients ak.
n z
n
n

Ratio test: Let lim


n

an +1
= L and R = 1/ L
an

a ( z z0 )
z
L ' = lim n +1 = lim n +1
n
n z
n
an ( z z0 )
n

n +1

= z z0 lim

z z0
an +1
= z z0 L =
an
R

n a = L and R = 1/ L
Root test: lim
n
n

L ' = lim n zn = lim n an ( z z0 ) = z z0 lim n an = z z0 L =


n

z z0
R

L ' < 1, z z0 < R , the power series converges


L ' > 1, z z0 > R , the power series diverges

L 0, radius of convergence R = 1/L


L = 0, radius of convergence R =
L = , radius of convergence R = 0

132

harmonic series

Example 5

Circle of Convergence

Consider the power series


Sol:

1
1
1 1
1 1 1 1
= 1+
+ + + + + + + ......
2
3 4
5 6 7 8
k =1 k

k =1

1
1 1
1 1 1 1
+ + + + + + + ......
2
4 4
8 8 8 8
1 1
= 1 + + + ......
2 2

z k +1
.
k

1
1
L = lim n + 1 = 1 =
n
1
R
n

> 1+

radius of convergence R = 1

Series absolutely converges for |z| < 1

Consider |z| = 1

1
diverges,
At z = 1,
k =1 k

( 1)

At z = 1,

k =1

k =1

k +1

: converges (alternating harmonic series)

( 1)

k =1

z k +1
k

k +1

k =1

z k +1
not converges
k

1 1 1
= 1 + ...... = ln 2 = 0.6931471806
2 3 4

converges at all points on |z| =1 except z = 1

133

Example 6

Radius of Convergence
rId5

Consider the power series


z0 = 1 + i

lim

ak

an +1
= lim
n
an

( 1)
=

k +1

k!
n +1
( 1)
( n + 1)!

( 1)

= lim

1
1
=0= , R=
n +1
R

Series converges for all z

n!

Example 7

Radius of Convergence
6k + 1
2k + 5

Consider the power series


z0 = 2i

6k + 1
ak =
2k + 5

k =1

( z 2i )

lim n an = lim
n

Series converges absolutely, z 2i <


Series diverges, z 2i >

1
3

Series inconclusive, z 2i =

1
3

6n + 1
1
1
=3= , R =
2n + 5
R
3

1
3

134

Contents
19.1 Sequences and Series
19.2 Taylor Series
19.3 Laurent Series
19.4 Zeros and Poles
19.5 Residues and Residue Theorem
19.6 Evaluation of Real Integrals
19.7 Summation of Series by the Residue Theorem
19.8 Inverse of Laplace Transforms

135

Properties of Power Series


Power series

ak ( z z0 ) has a radius of convergence R


k

k =0

Continuity

THEOREM 19.6

A power series

ak ( z z0 ) represents a continuous function f within its circle of


k

k =0

convergence |z z0| = R, R 0

k =0

ak ( z z0 ) = f ( z ) , z z0 < R, R 0
k

Term-by-Term Integration

THEOREM 19.7

A power series

proof in Churchill (uniformly convergence)


continuous can be replaced by analytic

k =0

ak ( z z0 ) can be integrated term by term within its circle of


k

convergence |z z0| = R, R 0, for every contour C lying entirely within the circle
of convergence
C

f ( z ) dz =

k =0

ak ( z z0 ) dz =
k

k =0

ak

( z z0 )
C

dz , C within z z0 < R
proof in Churchill
136

Term-by-Term Differentiation

THEOREM 19.8

A power series

k =0

ak ( z z0 ) can be differentiated term by term within its circle of


k

convergence |z z0| = R, R 0

d
d
k
k
ak ( z z0 ) = ak ( z z0 ) =
dz k =0
dz
k =0

f '( z ) =

kak ( z z0 ) , z z0 < R
k 1

k =1

proof in Churchill

Taylor Series

k =0

ak ( z z0 ) = f ( z ) , z z0 < R, then ak = ?
k

f ( z) =
f '( z ) =

Th. 19.8

, z z0 < R

ak k ( z z0 )

, z z0 < R

k =1

f '' ( z ) =

k =2

ak ( z z0 ) = a0 + a1 ( z z0 ) + a2 ( z z0 ) + a3 ( z z0 ) + ...
2

k =0

Note: R

k 1

= a1 + 2a2 ( z z0 ) + 3a3 ( z z0 ) + ...

ak k ( k 1)( z z0 )

k 2

= 2 1a2 + 3 2a3 ( z z0 ) + 4 3a4 ( z z0 ) + ..., z z0 < R


2

137

f ''' ( z ) =

z = z0

k =0

ak k ( k 1)( k 3)( z z0 )

f ( z0 ) = a0

ak =

f(

f ( z) =

f ( z) =

k)

f ' ( z0 ) = 1!a1

k 3

= 3 2 1a3 + 4 3 2a4 ( z z0 ) + ,... z z0 < R

f '' ( z0 ) = 2!a2

(k )
f ''' ( z0 ) = 3!a3 f ( z0 ) = k !ak

( z0 ) ,

k!

k =0

k =0

( k = 0, 1, 2, ...) : Taylor coefficients of f at z0


k
f ( ) ( z0 )
k
( z z0 ) : Taylor series of f (centered) at z0
k!

f (k ) ( 0) k
z : Maclaurin series
k!

Given

k =0

A Taylor series with center z0 = 0

k
ak ( z z0 ) = f ( z ) analytic, z z0 < R, ak =

f ( k ) ( z0 )
k!

Any power series defining a function f in |z z0| < R must be the Taylor series
of f about z0
Given f ( z ) analytic in D
f ( z) =

k =0

ak ( z z0 ) , z z0 < R, ak = ?, R = ?
k

138

Taylors Theorem

THEOREM 19.9

Let f be analytic within a domain D and let z0 be a point in D. Then f has the
series representation
f ( z) =

f(

k)

( z0 )

k!

k =0

( z z0 )

valid for the largest circle C with center at z0 and radius R that lies entirely in D.
Proof: C = {s s z = R} = the largest circle with center z and
0
0
radius R that lies entirely within D
z: fixed point inside C
s: variable point on C
1
1
1
1
=
=
z
s z ( s z 0 ) ( z z 0 ) s z 0 1 z0
s z0

z z0
<1
=
s z0

1
s z0

k =0

z z0
s z0

( z z0 )
=
k +1
k = 0 ( s z0 )
k

1
= 1 + z + z 2 + z 3 + ..., z < 1
1 z

f(s): analytic on and within C


1
f ( z) =
2 i
=

k =0

k =0

f (s)
1
ds =
2 i
sz

1
2 i
f(

k)

( z0 )

k!

f (s)

CIF

( z z0 )
f (s)
k +1
C
k = 0 ( s z0 )

( s z0 )
k

( z z0 )

139

k +1

ds

( z z0 )

f ( k ) ( z0 ) =

k!
2 i

ds

Term-by-Term Integration
f (s)

( s z0 )

k +1

ds

Note:
R = radius of convergence
= the radius of the largest circle C with center z0 that lies entirely within D
= |zs z0|
zs = the nearest isolated singularity at which f (z) is not analytic
1
analytic at zs = i, z0 = 0
1+ z2

1
k
2 k
f ( z) =
=

z
=
( 1) z 2 k , z < R = zs z0 = i 0 = 1
(
)
2
1+ z
k =0
k =0

[Ex] f ( z ) =

z <1
2

Isolated singularity: A point z at which the function f fails to be analytic,


but is analytic at all other points through out some N(z).

z s = +i
R =1

z0 = 0

zs = i

140

Note:
If two power series

k =0

k =0

ak ( z z 0 )
bk ( z z0 )

= f ( z),

z z0 < R

same function, if the center, radius of


convergence are the same.

ak = bk , k = 0, 1, 2, ...

Power series expansion of a function is unique


Taylor series obtained by different methods should be the same

Taylor Series Expansion of f(z)


By Taylor coefficients
f ( z ) = e z , f ( k ) ( z ) = e z , z0 = 0, f ( k ) ( 0 ) = 1
ez = f ( z ) =

k =0

f (k ) ( 0) k z k
z =
k!
k =0 k !

z <R=

e z = f ( z ) entire function
141

By analytic connection

xz

Real Taylor series

sin x =

k =0

( 1)

Complex Taylor series

x 2 k +1
, x <
( 2k + 1)!

xz

z 2 k +1
, z <
( 2k + 1)!
k =0
2k

k z
cos z = ( 1)
, z <
( 2k ) !
k =0

sin z =

( 1)

By known series
f ( z) =

(1 z )

z0 = 0

1
g ( z) =
= zk ,
1 z k =0

z <1

diff. term by term


g '( z ) =

(1 z )

= f ( z) =

kz k 1 ,

z <1

k =1

By multiplication or division of known series


tan z =

k =0

sin z = z

ak z k

ak = ?

z3 z5 z7
+ + ...
3! 5! 7!

cos z = 1

z2 z4 z6
+ + ...
2! 4! 6!
142

sin z
sin z = ( tan z )( cos z )
cos z
z3 z5 z7
z2 z4 z6
z + + ... = ( a0 + a1 z + a2 z 2 + a3 z 3 + a4 z 4 + a5 z 5 + a6 z 6 + ...) 1 + + ...
3! 5! 7!
2! 4! 6!
tan z =

= a0 + a1 z +
+

a0 = 0
a1 = 1
a
0 + a2 = 0
2!
a0 a2
+ a4 = 0
4! 2!

a0
a a
a
+ a2 z 2 + 1 + a3 z 3 + 0 2 + a4 z 4
2!
2!
4! 2!

a a a
a1 a3
+ a5 z 5 + 0 + 2 4 + a6 z 6 + ...
4! 3!
6! 4! 2!
a1
1
+ a3 =
2!
3!
a1 a3
1
+ a5 =
4! 3!
120

a2 = 0

a4 = 0

a0 a2 a4
+ + a6 = 0
6! 4! 2!
1
2
tan z = z + z 3 + z 5 + ...
3
15

a3 =

1
3

a5 =

2
15

a6 = 0

z < R = z s z0 =

tan z =

0 =

sin z

analytic at zs = ,...
cos z
2
143

Example 2

Taylor Series

Expand f ( z ) =
Sol:

1
in a Taylor series with center z0 = 2i
1 z

(1) by Taylor coefficients


f '( z ) =
f(

k)

(1 z )

( 2i ) =

f '' ( z ) =

2 1

(1 z )

f ''' ( z ) =

(1 z )

f(

k)

( z) =

k!

(1 z )

k +1

k!

(1 2i )

k +1

f ( ) ( 2i )
( z 2i )
1
k
f ( z) =
=
( z 2i ) =
k +1
1 z k =0 k !
k = 0 (1 2i )

f (z) =

3 2 1

1
analytic at zs = 1
1 z

z 2i < R = 5

R = zs z0 = 1 2i = 5

z0 = 2i

(2) by known series

1
= tk
1 t k =0

t <1

1
1
=
=
1 z (1 2i ) ( z 2i )

zs = 1
1

(1 2i )

z 2i
1 2i

144

t=

z 2i
z 2i
< 1, z 2i < 1 2i = 5
, t =
1 2i
1 2i

1
1
=
=
1 z (1 2i ) ( z 2i )
1 z 2i
=
1 2i k =0 1 2i

(1 2i )

z 2i
1 2i

( z 2i )
=
k +1
k = 0 (1 2i )

z 2i < 5

Note:
f ( z) =

1
= z k = ak z k ,
1 z k =0
k =0

z <1

1
1
k
k
=
z

i
=
bk ( z 2i ) ,
2
(
)
1 z k =0 (1 2i )k +1
k =0

z 2i < 5

z * shaded region

ak ( z * ) =
k

k =0

k =0

bk ( z * 2i ) =
k

1
= f ( z* )
1 z*

Outside of shaded region, at least one of the two series must diverge
145

Homework 5
19.1
10, 14, 28, 29

19.2
12, 26, 32, 34

146

Contents
19.1 Sequences and Series
19.2 Taylor Series
19.3 Laurent Series
19.4 Zeros and Poles
19.5 Residues and Residue Theorem
19.6 Evaluation of Real Integrals
19.7 Summation of Series by the Residue Theorem
19.8 Inverse of Laplace Transforms

147

Singularities
Def.
z0 singularity or singular point of f

f analytic at z0

f analytic at some point in every neighborhood N ( z0 ) = z 0 < z z0 < of z0

Def.
z0 isolated singularity of f

f analytic at z0

f analytic in N ( z0 ) = z 0 < z z0 <

}
y

[Ex.]
f ( z) =

z
z
=
analytic everywhere except at z = 2i
z + 4 ( z + 2i )( z 2i )

z = 2i

z = 2i : isolated singularity of f

f analytic in 0 < z 2i < 1

f analytic in 0 < z ( 2i ) < 1

x
z = 2i

148

Def.
z0 nonisolated singularity of f
f analytic at z0

and every N(z0) contains at least one singularity of f other than z0


[Ex.]

f ( z ) = Ln z analytic in D = { z z 0, < Arg z < }

Non-positive real axis = { z Im z = 0, Re z 0}: nonisolated singularity of Ln z


y

Ex:
f (z) = 1/z
f (z) = Ln z
f (z) = 1/sin(/z), z = 1/n, n = integers
f (z) = |z|2 has no singularities.

149

A New Kind of Series


z0 isolated singularity of f
f ( z) =

k =1

a k

( z z0 )

Principal part:

k =0

f ( z) =

k =

ak ( z z0 ) =
k

k =0

k =1

Analytic part:

a k

( z z0 )

k =

k
ak ( z z0 ) : Laurent series (or Laurent

converges,

expansion) of f

1
1
< r * or z z0 > * = r
z z0
r

ak ( z z0 ) converges, z z0 < R
k

ak ( z z0 ) converges, r < z z0 < R


k

Example 1

A New Kind of Series

sin z
analytic at z = 0 cannot be expanded in a Maclaurin (or Taylor)
z3
series
z3 z5 z7
sin z = z + + ......, z <
3! 5! 7!
f ( z) =

f ( z) =

sin z 1 1 z 2 z 4
= 2 + + ......, 0 < z <
z3
z 3! 5! 7!

Laurent series
150

Laurents Theorem

THEOREM 19.10

Let f be analytic within the annular domain D defined by r < |z z0| < R. Then f has
the series representation
f ( z) =

k =

ak ( z z0 )

Laurent series of f about z0

valid for r < |z z0| < R. The coefficients ak are given by


ak =

f (s)

1
2 i

( s z0 )

k +1

ds, k = 0, 1, 2, ......, Laurent coefficients of f about z0

where C is a simple closed curve that lies entirely within D and has z0 in its interior.

Recall: CIFD for Taylors thm


ak = f (k) (z0)/k! ( k=0, 1, 2...)

151

Proof:

{
= {s s z

C1 = s s z0 = r1
C2

= R2

r < r1 < R2 < R

R2

z: fixed point in D st. r1 < z z0 < R2

r1

s: variable point on C1 and C2


Cuts, K = C2 + cuts + C1
f analytic on and within K

CIF

f (z) =

1
2 i

f ( z) =

1
2 i

For s on C2

C2

f (s)
1
ds =
2 i
sz
f (s)
1
ds
2 i
sz

z z0
s z0

C2

C1

f (s)
ds +
sz

f (s)
ds
sz

cuts

f (s)
ds +
sz

C1

f (s)
ds
sz

=0

<1

1
1
1
1
1
=
=
=
s z ( s z0 ) ( z z0 ) s z0 1 z z0 s z0
s z0

k =0

z z0
s z0

( z z0 )
=
k +1
k = 0 ( s z0 )

152

1
2 i

C2

f (s)
1
ds =
2 i
sz
=

1
2 i

C2

f (s)

C2

deform

( s z0 )

f ( s ) ds

f (s)

1
2 i

k =0

ak =

( z z0 )
k +1
k = 0 ( s z0 )

C2

( s z0 )

ds
k +1

( z z0 )

k =0

ak ( z z0 )

ds, k = 0, 1, 2,......

k +1

s z0

<1

z z0

1
1
1
1
1
=
=
=
s z ( s z0 ) ( z z 0 ) z z0 1 s z0 z z0
z z0
C1

For s on C1

1
2 i

f (s)
1
ds =
2 i
sz
=

C1

1
2 i

m =1

( s z0 )
m
m =1 ( z z0 )

m 1

C1

k =0

s z0
z z0

m = k +1

( s z0 ) = ( s z0 )
=
k +1
m
k = 0 ( z z0 )
m =1 ( z z0 )
k

m 1

f ( s ) ds

f ( s )( s z0 )

m 1

( z z0 )

ds

m =1

a m ( z z 0 )

153

a m =
k = m

ak =

1
2 i

1
2 i

deform

C1

f ( s )( s z0 )
f (s)

C1

( s z0 )

k +1

m 1

ds =

1
2 i

f (s)

C1

( s z0 )

m +1

ds, m = 1, 2, 3, ...

ds, k = 1, 2, 3, ...

154

Note:
(1) f ( z ) =

k =

f1 ( z ) =
f2 ( z ) =

ak ( z z0 ) =

k =0

ak ( z z0 ) +
k

m =1

a m

( z z0 )

= f1 ( z ) + f 2 ( z ) , r < z z0 < R

ak ( z z0 ) analytic at z0, Taylor series about z0


k

k =0

a m

m =1

( z z0 )

Note here r can be 0


and R can be infinite

analytic at z0

(2) a. No matter how a Laurent expansion of f is obtained in D, the Laurent series


obtained is unique
b. The formula ak =

1
2 i

f (z)

( z z0 )

k +1

dz is seldom used in finding the Laurent series

c. Known series (Taylor or Geometric series)


(3) f ( z ) =
C

1
k
ak ( z z0 ) , ak =
2 i
k =

f (s)

( s z0 )

k +1

Laurent series

ds, k = 0, 1, 2, ......

f ( z ) dz = 2 ia1 : evaluation of integrals

155

Example 2

Laurent Expansions

Expand f ( z ) =

1
in Laurent series valid for (a) 0 < |z| < 1,
z ( z 1)

(b) 1 < |z|, (c) 0 < |z 1| < 1, and (d) 1 < |z 1|


Sol: (a) 0 < z < 1, z0 = 0
f ( z) =

1 1
1
1
= (1 + z + z 2 + ...) = 1 z z 2 ..., 0 < z < 1
z 1 z
z
z

1
< 1, z0 = 0
z
1 1
1
1
1
f ( z) = 2
= 2 1+ +
1
z 1
z
z
z
z

(b) 1 < z ,

+ ... =

1 1 1
+ + + ..., 1 < z
z 2 z3 z 4

(c) 0 < z 1 < 1, z0 = 1


f ( z) =
=

1
1
1
2
3
=
1 ( z 1) + ( z 1) ( z 1) + ...
z 11 + ( z 1) z 1
1
2
1 + ( z 1) ( z 1) + ..., 0 < z 1 < 1
z 1

156

1
< 1, z0 = 1
z 1
1
1
1
=
f ( z) =
z 1 ( z 1) + 1 ( z 1)2

(d) 1 < z 1 ,

1
1
1
+
z 1
z 1

=
2
1
( z 1)
1+
z 1
1
1
1
1
=

+ ..., 1 < z 1
2
3
4
5
( z 1) ( z 1) ( z 1) ( z 1)

Example 3

z 1

+ ...

Laurent Expansions

Expand f ( z ) =

( z 1) ( z 3)
2

in Laurent series valid for (a) 0 < |z 1| < 2,

(b) 0 < |z 3| < 2.

Sol: (a) 0 < z 1 < 2, z = 1


0

z 1
<1
2

1
1
1
1
z 1
z 1
=
=
+
1+
2
2
2

1
z
2
2
2 ( z 1)
( z 1) 2 + ( z 1) 2 ( z 1) 1
2
1
1
1 1
=

( z 1) ..., 0 < z 1 < 2


2
2 ( z 1) 4 ( z 1) 8 16
1

f ( z) =

z 1
2

+ ...

157

(b) 0 < z 3 < 2, z0 = 3


f ( z) =

(1 + t )

= 1 + ( 2 ) t +

1
1
1
=
2
( z 3) ( 2 + z 3) 4 ( z 3)

( 2 )( 3) t 2 + ( 2 )( 3)( 4 ) t 3 + ..., t
2!

1
1+

z 3
2

z 3
z 3
1
+3
1 2
4 ( z 3)
2
2

( z 3) + ..., 0 < z 3 < 2


1
1 3
+ ( z 3)
4 ( z 3) 4 16
4

<1

General binomial thm.

z 3
2

3!

+ ...

Example 4

A Laurent Expansion

Expand f ( z ) =
Sol:

8z + 1
in Laurent series valid for 0 < |z| < 1
z (1 z )

8z + 1 1
1
= 8 + (1 + z + z 2 + z 3 + ...)
z 1 z
z
1
= + 9 + 9 z + 9 z 2 + ..., 0 < z < 1
z

f ( z) =

158

Example 5

Expand f ( z ) =
Sol:

Note
z0 = 2 is an analytic point of f (z)

A Laurent Expansion

1
in Laurent series valid for 1 < |z 2| < 2
z ( z 1)

f ( z) =

1
1
1
=
+
= f1 ( z ) + f 2 ( z )
z ( z 1) z z 1

f1 ( z ) =

1
1
1
=
=
z 2 + ( z 2) 2

z2
<1
2

1
z2
z2
1
=
+
1
z2
2
2
2
1+
2

z2
2

+ ...

1 z 2 ( z 2) ( z 2)
= + 2
+
..., z 2 < 2
2
2
23
24
2

1
1
1
f2 ( z ) =
=
=
z 1 ( z 2) + 1 z 2
1
1
1
=

+
2
z 2 ( z 2 ) ( z 2 )3

1
1
1
=
+
1
z2
z2
z2

1
1+
z2
1

+ ..., 1 < z 2
4
( z 2)

z2

+ ...

1
<1
z2

f ( z ) = f1 ( z ) + f 2 ( z ) 1 < z 2 < 2
159

Example 6

A Laurent Expansion

Expand f ( z ) = e3/ z in Laurent series valid for 0 < |z|.


Sol:

t2 t3
+ + ..., t <
2! 3!
1
3 3
<
0< z
t= ,
<
z
z z
3 32
33
f ( z ) = e3/ z = 1 + +
+
+ ..., 0 < z
z 2! z 2 3! z 3
et = 1 + t +

160

Contents
19.1 Sequences and Series
19.2 Taylor Series
19.3 Laurent Series
19.4 Zeros and Poles
19.5 Residues and Residue Theorem
19.6 Evaluation of Real Integrals
19.7 Summation of Series by the Residue Theorem
19.8 Inverse of Laplace Transforms

161

z0 isolated singularity of f

Suppose f ( z ) =

k =1

a k

( z z0 )

k =

ak ( z z0 ) =
k

k =1

a k

( z z0 )

k =0

ak ( z z0 ) , 0 < z z0 < R
k

Punctured open disk

: principal part of f at z0
Note here is a special case of
Laurent series for r = 0

Classification of Isolated Singular Points


Removable singularity

a n 0
a k = 0, k > n

Pole of order n

Simple pole

all a-k = 0

principal part = 0
principal part contains finite
number of nonzero terms

all
pole of order 1 a1 0, a k = 0 for k > 1

Essential singularity

principal part contains infinite


number of nonzero terms

162

z = z0

Laurent Series
a0 + a1 ( z z0 ) + a2 ( z z0 ) + ......
2

Removable singularity

a n

Pole of order n

( z z0 )

Essential singularity

f ( z) =

a( n 1)

( z z0 )

n 1

+ ... +

a1
2
+ a0 + a1 ( z z0 ) + a2 ( z z0 ) + ......
z z0

a1
2
+ a0 + a1 ( z z0 ) + a2 ( z z0 ) + ......
z z0
a2
a
2
...... +
+ 1 + a0 + a1 ( z z0 ) + a2 ( z z0 ) + ......
2
( z z0 ) z z 0

Simple pole

Example 1

Removable Discontinuity

sin z 1
z3 z5
z2 z4
=
z + ...... = 1 + ......, z <
z
z
3! 5!
3! 5!

z = 0 : removable singularity of

sin z
z

Rigorously speaking the range of z


should be 0 < | z | < .
If f (0) = a0 = 1, we can denote
|z|<.
163

Example 2

Poles and Essential Singularity

3
5
3
(a) f ( z ) = sin2 z = 12 z z + z ...... = 1 z + z ......, 0 < z <

3!

5!

3! 5!

z = 0 : simple pole
f ( z) =

sin z 1
z3 z5
1 1 z2
=
z

......
=
+ ......, 0 < z <
z3
z3
3! 5!
z 2 3! 5!

z = 0 : pole of order 2

(b) f ( z ) =
z =1

( z 1) ( z 3)
2

1
2 ( z 1)

1
1 z 1

......, 0 < z 1 < 2
4 ( z 1) 8 16

: pole of order 2
3

3
z

(c) f ( z ) = e z = 1 + +

32
33
+
+ ......, 0 < z <
2! z 2 3! z 3

z = 0 : essential singularity

164

Example

Punctured open disk

1
1 z z 2 ......, 0 < z < 1 ...(a)
1
z
=
f ( z) =
1 1 1
z ( z 1)
+ + + ......, 1 < z ............(b)
z 2 z3 z 4
z = 0 isolated singularity of f
z = 0 simple pole

(a)
z = 0 essential singularity

(b) z > 1: not a punctured open disk

Zeros
f ( z0 ) = 0

[Def.] z0: zero of f

f ( z0 ) = f ' ( z0 ) = ... = f (

[Def.] z0: zero of order n of f

n 1)

( z0 ) = 0

f ( n ) ( z0 ) 0
f ( z ) = an ( z z0 ) + an +1 ( z z0 )
n

= ( z z0 )

n +1

+ an + 2 ( z z 0 )

n+2

+ ...... a 0
n

an + an +1 ( z z0 ) + an + 2 ( z z0 ) + ......
2

= ( z z0 ) g ( z )
n

165

Example
f ( z ) = ( z 5)

Example 3

z = 5 : zero of order 3

f (5) = f (5) = f (5) = 0,


f (5) = 6

Order of a Zero

f ( z ) = z sin z = z z
2

(z ) + (z )

2 3

= z3 1

3!

2 5

5!

......

z 4 z8
+ ...... , z <
3! 5!

Note:
1. f nontrivial analytic function
2. z0 : zero of f
z0 : isolated zero

z = 0 : zero of order 3

Note: Ex. f (z) = sin (/z)


z = 0 is not an isolated zero, but an essential
singularity
z = 1/n, for non-zero integer n, is an isolated zero

f ( z0 ) = 0 ; but f ( z ) 0, z N ( z0 ) = z 0 < z z0 <

Note:
z0: zero of nontrivial analytic function f
F ( z) =

1
has isolated singularity at z0
f ( z)

166

THEOREM 19.11

Pole of Order n

If the functions f and g are analytic at z = z0 and f has a zero of order n at z = z0


and g(z0) 0, then the function F(z) = g(z)/f (z) has a pole of order n at z = z0
Poles of

g (z)

Zeros of f ( z ) Proof in Churchill

f ( z)

Example 4
F ( z) =

Order of Poles

2z + 5

( z 1)( z + 5)( z 2 )
4
f ( z ) = ( z 1)( z + 5 )( z 2 )

zeros of order 1 at z = 1
zeros of order 1 at z = 5
zeros of order 4 at z = 2

g ( z)

f ( z)

Ex. 5. f (z) = z sin z2


z = 0, zero of order 3
Ex. 6. f (z) = 1 / (z sin z2)
z = 0, pole of order 3
g ( z0 ) 0, z0 = 1, 5, 2
F ( z ) has simple pole at z = 1 and z = 5

pole of order 4 at z = 2

167

Homework 6
19.3
4, 12, 20, 28

19.4
10, 16, 24

168

Contents
19.1 Sequences and Series
19.2 Taylor Series
19.3 Laurent Series
19.4 Zeros and Poles
19.5 Residues and Residue Theorem
19.6 Evaluation of Real Integrals
19.7 Summation of Series by the Residue Theorem
19.8 Inverse of Laplace Transforms

169

Residue

Note here r (inner radius of the domain D)


must be 0, otherwise the statement is wrong!!

z0 isolated singularity of f

f analytic at z0

f analytic in D = z 0 < z z0 < R


f ( z) =

k =

f ( z) =

ak ( z z0 ) , 0 < z z0 < R
k

k =2

a k

( z z0 )

a1
k
+ ak ( z z0 ) , 0 < z z0 < R
z z0 k = 0

Let C: simple closed contour in D enclosing z0


C

f ( z ) dz =

k =2

a k

( z z0 )

dz + a1

2 i

0
a1 =

1
2 i

f ( z ) dz

k = 1

f ( z ) dz = 2 ia1 f ( z ) , z0

1
dz + ak
z z0
k =0

ak =

1
2 i

( z z0 )

dz = 2 ia1

0
f (z)

( z z0 )

k +1

Recall CIF:

dz

f ( z0 ) =

1
2 i

f ( z)
dz
z z0
170

[Def.] Residue of f (z) at z0 = Res f ( z ) , z0 a1 f ( z ) , z0


= coefficient of

Example 1

Residue

(a) f ( z ) =

( z 1) ( z 3)

1
in Laurent expansion of f at z0
z z0

in a punctured disk

1
2 ( z 1)

1
1 1
( z 1) ......, 0 < z 1 < 2
4 ( z 1) 8 16

z = 1: pole of order 2
Res f ( z ) ,1 =

1
4

3
2
(b) f ( z ) = e z = 1 + 3 + 3 2 + ......, 0 < z <

2! z

z = 0: essential singularity
Res f ( z ) , 0 = 3

171

f ( z) =

a1
2
+ a0 + a1 ( z z0 ) + a2 ( z z0 ) + ......
z z0

Residue at a Simple Pole

THEOREM 19.12

If f has a simple pole at z = z0, then


Res f ( z ) , z0 = lim ( z z0 ) f ( z )
z z0

Residue at a Pole of Order n

THEOREM 19.13

If f has a pole of order n at z = z0, then


Res f ( z ) , z0 =

1
d n 1
lim
( n 1)! z z0 dz n1

Proof: f has a pole of order n


f ( z) =

a n

+ ... +

a2

( z z0 )

f ( z)

a1
+ a0 + a1 ( z z0 ) + ...
z z0

( z z0 )
( z z0 )
n
n2
n 1
n
n +1
g ( z ) ( z z0 ) f ( z ) = a n + ... + a2 ( z z0 ) + a1 ( z z0 ) + a0 ( z z0 ) + a1 ( z z0 ) + ...
n 1

analytic at z0

d
g ( z ) = ( n 1) !a1 + n !a0 ( z z0 ) + ...
dz n 1
d n 1
d n 1
n
lim n 1 g ( z ) = lim n 1 ( z z0 ) f ( z ) = ( n 1) !a1 = ( n 1) !Res f ( z ) , z0
z z0 dz
z z0 dz
1
d n 1
n
n = 1 for Th. 19.12
Res f ( z ) , z0 =
lim
z z0 ) f ( z )
n 1 (
z
z

0
dz
( n 1)!

172

Note:
1. f ( z ) =

g ( z)
h( z)

If h(z) has zeros of order n,


the formula is complicated.

2. g ( z ) & h ( z ) analytic at z0

3. g ( z0 ) 0 & h ( z ) has simple zero at z0 h ( z0 ) = 0, but h ' ( z0 ) 0


f(z) has a simple pole at z0 &
Res f ( z ) , z0 = Res

g ( z)
g ( z0 )
, z0 =
h( z)
h ' ( z0 )

= lim
z z0

Note: Determination of Residues


Let z0: isolated singularity of f (z)

Res f ( z ) , z0 =

2. z0: pole of order n

Example 2
f ( z) =

1
d n 1
lim
( n 1)! z z0 dz n1

Res f ( z ) , z0 = lim ( z z0 ) f ( z )

f ( z)

g (z)
g ( z0 )
, z0 =
h( z)
h ' ( z0 )

173

Residue at a Pole
1

( z 1) ( z 3)
2

Res f ( z ) ,3 = lim ( z 3) f ( z ) = lim


z 3

z = 1 : pole of order 2

Res f ( z ) ,1 =

z 3

2 1

1
d
lim
( 2 1)! z 1 dz 21

= lim
z 1

Example 3

( z z0 )

Res

z z0

z = 3 : simple pole

f ( z) =

g ( z)
g ( z0 )
=
h ( z ) h ( z0 ) h ' ( z 0 )
z z0

Res f ( z ) , z0 = a1 f ( z ) , z0
1
= coefficient of
in Laurent expansion of f at z0
z z0

1. z0 : essential singularity

3. z0: simple pole

g ( z)
g ( z)
, z0 = lim ( z z0 )
z z0
h( z)
h( z)

Res

( z 1)

( z 1)

1
4

f ( z)

d 1
1
=
dz z 3
4

g ( z)
1
1
= 4
=
h ( z ) z + 1 ( z z1 )( z z2 )( z z3 )( z z4 )

z1 = e 4 , z2 = e

3 i
4

, z3 = e

5 i
4

, z4 = e

7 i
4

: simple poles

1
1
g (zj )
g ( z)
Res f ( z ) , z2 =

i
1
= 3 , j = 1, 2, 3, 4
Res
, zj =
4 2 4 2
h( z)
h '( z j ) 4z j
1
1
Res f ( z ) , z3 =
+
i
1
1
1
4 2 4 2
Res f ( z ) , z1 =
=

i
1
1
i 3
4 2 4 2
+
i
Res f ( z ) , z4 =
4 e4
4 2 4 2

174

Residue Theorem
Cauchys Residue Theorem

THEOREM 19.14

Let D be a simply connected domain and C a simple closed contour lying entirely
within D. If a function f is analytic on and within C, except at a finite number of
isolated singularities z1, z2, , zn within C, then
C

f ( z ) dz = 2 i

n
k =1

Res f ( z ) , zk

Proof:

Ck = z z zk = rk = circle of radius rk centered at zk

zk

st. C1, C2, , Cn are mutual disjoint and are interior to C ( k = 1, 2, ..., n )
Cuts, K = C + Cuts +

n
k =1

0=
C

f ( z ) dz =

f ( z ) dz =

f ( z ) dz = 2 i

n
k =1

Example 4

f ( z ) analytic on and within K

( Ck )
Cuts

n
k =1

f ( z ) dz =

Ck

k =1

Ck

Ck

k =1

f ( z ) dz =

n
k =1

f ( z ) dz

Ck

2 ia1 f ( z ) , zk

Or direct consequence from CGT


for multiply connected domain

Res f ( z ) , zk

175

Evaluation by the Residue Theorem


1

Evaluate

( z 1) ( z 3)
2

dz , where

(a) the contour C is the rectangular defined by x = 0, x = 4, y = 1, y = 1, and (b)


the contour C is the circle |z| = 2
Sol:

(a) Both poles z = 1 and z = 3 lie within C


1
C

( z 1) ( z 3)
2

dz = 2 i Res f ( z ) ,1 + Res f ( z ) ,3

Res f ( z ) ,3 = lim ( z 3) f ( z ) = lim


z 3

Res f ( z ) ,1 =

z 3

2 1

1
d
lim
( 2 1)! z 1 dz 21

( z 1)

( z 1)

f ( z)

}=0

1
4

( z 1) ( z 3)
2

dz = 2 iRes f ( z ) ,1 =

= lim
z 1

d 1
1
=
4
dz z 3

(b) Only the pole z = 1 lies within C


1

i
1

176

Example 5

Evaluation by the Residue Theorem

2 z + 6 , where the contour C is the circle |z i| = 2


dz
C z2 + 4
2z + 6
2z + 6
=
f ( z) = 2
z + 4 ( z + 2i )( z 2i )

Evaluate
Sol:

C
2i
i
x

z = 2i : simple pole and within C


z = 2i : simple pole and not within C
2z + 6
dz = 2 iRes f ( z ) , 2i = 2 i lim
C z2 + 4
z 2i

Example 6

2i

( z 2i )

2z + 6
z2 + 4

= 2 i

3 + 2i
= ( 3 + 2i )
2i

Evaluation by the Residue Theorem

ez
dz , where the contour C is the circle |z| = 2
C z 4 + 5z3
z = 0 : pole of order 3 and within C
ez
ez
f ( z) = 4
=
3
3
z + 5z
z ( z + 5)
z = 5 : simple pole and not within C

Evaluate

Sol:

ez
dz = 2 iRes f ( z ) , 0 = 17 i
C z 4 + 5z3
125
1
d2 3
ez
17
Res f ( z ) , 0 = lim 2 z
=
3
2! z 0 dz
250
z ( z + 5)

Example 7

Evaluation by the Residue Theorem

Evaluate
Sol:

3
5
simple poles at , , , ...

sin z
cos z

tan zdz = 2 i Res f ( z ) ,

Res f ( z ) =

g ( z)
h( z)

Example 8

, z0 =

g ( z0 )

h ' ( z0 )

within C

+ Res f ( z ) ,
=

= 4 i

sin z0
= 1
sin z0

Evaluation by the Residue Theorem

Evaluate
Sol:

tan zdz, where the contour C is the circle |z| = 2

f ( z ) = tan z =

177

e z dz , where the contour C is the circle |z| = 1

3
z

3 32
f ( z) = e = 1+ +
+ ..., 0 < z <
z 2! z 2
z = 0 : essential singularity and within C
3

e z dz = 2 iRes f ( z ) , 0 = 2 ia 1 f ( z ) , 0 = 2 i 3 = 6 i
178

Example

sin 2 z

Evaluate
Sol:

(1) f ( z ) =

( z i)

dz, where the contour C is the circle |z i| =1

sin 2 z

( z i)

z = i : pole of order 3 and within C

Res f ( z ) , i =

1
d2
lim 2
2! z i dz

( z i)

sin 2 z

( z i)

1
= lim ( 4sin 2 z ) = 2sin 2i = 2i sinh 2
2 z i

(2) sin 2 z = sin 2 ( z i + i ) = sin 2 ( z i ) cos 2i + cos 2 ( z i ) sin 2i


= cos 2i 2 ( z i )
f ( z) =

sin 2 z

( z i)

a3

( z i)

a1 = 2sin 2i

(3)

sin 2 z
C

( z i)

2( z i)
3!
a2

( z i)

+ ... + sin 2i 1

2( z i)
2!

+ ...

a1
+ a0 + a1 ( z i ) + ...
z i

Res f ( z ) , i = a1 = 2sin 2i

dz = 2 iRes f ( z ) , i = 2 i ( 2i sinh 2 ) = 4 sinh 2

179

L'Hopital's
rule
g ( z)
(1) f ( z ) =
; g ( z ) & h ( z ) analytic at z0
h( z)

(2) g ( z0 ) = h ( z0 ) = 0 but h ' ( z0 ) 0


lim

z z0

g ( z ) g ' ( z0 )
=
h ( z ) h ' ( z0 )

g ( z ) g ( z0 )
g ( z)
z z0
g '( z0 )
lim
= lim
=
z z0 h ( z )
z z0 h ( z ) h ( z )
h '( z0 )
0
z z0

Summary (Important!!)
Def. of Residue, notes p. 170
Methods to find residues, notes p. 172, 173
Cauchys Residue Theorem, notes p. 175
180

Contents
19.1 Sequences and Series
19.2 Taylor Series
19.3 Laurent Series
19.4 Zeros and Poles
19.5 Residues and Residue Theorem
19.6 Evaluation of Real Integrals
19.7 Summation of Series by the Residue Theorem
19.8 Inverse of Laplace Transforms

181

A.

F ( cos ,sin ) d

f (z) dz, C: | z | = 1

(1) z = ei , 0 2 , dz = iei d = izd , d =


1 i
1
1
e + e i ) =
z+
(
2
2
z
1
1
1
sin = ( ei e i ) =
z
2i
2i
z

dz
iz

cos =

(2) F ( cos ,sin ) = F

1
1 1
1
z+ ,
z
2
z 2i
z

Th.
2
0

F ( cos ,sin ) d =

f ( z) =

z =1

1
1 1
1
z+ ,
z
2
z 2i
z
iz

has no singularities on |z| = 1

1
1 1
1
z+ ,
z
2
z 2i
z

dz
= 2 i
iz

Res f ( z ) , zk

, zk: poles of f (z) within |z| = 1

182

Example 1

A Real Trigonometric Integral

( 2 + cos )

Sol:

Evaluate

( 2 + cos )

f ( z) =

d =

( z + 4 z + 1)
2

( 2 + cos )

2+

1
1
z+
2
z
z

( z z0 ) ( z z1 )
2

z1 = 2 + 3 within |z| = 1

d =

Res f ( z ) , z1 =

z =1

1
1
dz
z + , d =
2
z
iz
dz 4
z
=
dz
2
1
z
=
2
iz i
( z + 4 z + 1)

z = ei , cos =

z0 = 2 3

pole of order 2 at
2

4
1
4
2 iRes f ( z ) , z1 = 8
=
i
6 3 3 3

d
1
lim
1! z z1 dz

( z z1 )

d
z z1 dz

f ( z ) = lim

( z z0 )

= lim

z z1

( z + z0 ) = 1
3
( z z0 ) 6 3

183

Improper Integrals f ( x ) continuous for < x <

f ( x ) dx = lim

r r

f ( x ) dx + lim

R 0

f ( x ) dx = L1 + L2

Def.

f ( x ) dx convergent

both L1 and L2 exist

divergent

one or both of L1 and L2 fail to exist

Note:

(1)
f ( x ) dx convergent

(2)

f ( x ) dx = lim

f ( x ) dx = PV

R R

f ( x ) dx divergent

PV

Example

xdx divergent

lim

R 0

f ( x ) dx : Cauchy principal value

f ( x ) dx may exist

R2
=
R 2

xdx = lim

R2 ( R )
PV
xdx = lim
xdx = lim

R R
R
2
2

=0

If f ( x ) even function, f ( x ) = f ( x )
R

f ( x ) dx =

PV

1
2

f ( x ) dx = 2

f ( x ) dx

f ( x ) dx
184

B.

f ( x ) dx, f ( x ) =

(1) f ( x ) =

P ( x)
Q ( x)

P ( x)
= rational function of x
Q ( x)

P ( x ) , Q ( x ) : polynomial functions of x with real coefficients

(2) Q ( x ) 0 for all real x


f ( z) =

P( z)

Q( z)

has no real poles

zk ( k = 1, 2,..., n ) : poles of f (z) in upper half plane (UHP)


y

C = L + CR
R
R

f ( z ) dz +

CR

f ( z )dz =

PV

f ( z ) dz = 2 i

n
k =1

Res f ( z ) , zk

CR
zk

f ( x ) dx

Th. 19.15

185

THEOREM 19.15

Behavior of Integral as R

Suppose f(z) = P(z)/Q(z), where the degree of P(z) is n and the degree of Q(z) is
m n + 2. If CR is a semicircular contour z = Rei, 0 , then
CR

f ( z )dz 0 as R

Proof: deg Q ( z ) deg z 2 P ( z )


z P( z)
M
Q( z)
2

CR

f ( z )dz =

Th.
f ( z) =

z CR

deg Q(z) > 1 + deg P(z) will be enough


or m > n + 1, if m, n are non-integers

P( z) M M
2 = 2
Q( z)
R
z
P( z)
M
dz 2
CR Q z
R
( )

CR

dz =

M
M R
=

R
(
)
0
R2
R

P( z)

, P ( z ) , Q ( z ) : polynomial functions of z with real coefficients


Q( z)
deg Q ( z ) deg z 2 P ( z )
f ( z ) has no real poles (

Q ( z ) has no real zeros)

poles zk (k = 1, 2, , n) in UHP
PV

f ( x ) dx = 2 i

n
k =1

Res f ( z ) , zk

186

- f(z) = P(z)/Q(z): real coefficient


- deg[Q(x)] deg[z2P(x)]
- f(z) has no real poles

Example 2
Evaluate the Cauchy principal value of
Sol:

f ( z) =

(z

+ 1)( z + 9 )
2

(x

+ 1)( x 2 + 9 )

dx

1
( z + i )( z i )( z + 3i )( z 3i )

Simple poles: z = i, i, 3i, 3i


PV

(x

UHP

+ 1)( x 2 + 9 )

Res ( f , i ) = lim ( z i )
z i

dx = 2 i Res ( f , i ) + Res ( f ,3i )

1
1
=
( z + i )( z i )( z + 3i )( z 3i ) 16i

Res ( f ,3i ) = lim ( z 3i )


z 3i

PV

(x

+ 1)( x + 9 )
2

1
1
=
( z + i )( z i )( z + 3i )( z 3i ) 48i

dx =

12

187

Example 3
1
dx
x + 1
g ( z)
1
Sol: f ( z ) = 1 =
=
4
z + 1 ( z z1 )( z z2 )( z z3 )( z z4 ) h ( z ) - f(z) = P(z)/Q(z): real coefficient

Evaluate the Cauchy principal value of

3 i
4

5 i
4

7 i
4

Simple poles: z1 = e 4 , z2 = e , z3 = e , z4 = e ,

- deg[Q(x)] deg[z2P(x)]
- f(z) has no real poles

UHP
1
PV
dx = 2 i Res ( f , z1 ) + Res ( f , z2 )
x 4 + 1
g ( zk )
1
g
Res f = , zk =
= 3
h
h ' ( zk ) 4 zk

Res ( f , z1 ) =

4 2 4 2
1
1
Res ( f , z2 ) =
i

4 2 4 2

PV
dx =
4
x + 1
2

188

f ( x ) cos xdx

C.

f ( x ) sin xdx

f ( x ) e dx =
i x

Q ( x)

f ( x ) cos xdx + i

C = L + CR
R
R

f ( z ) e dz +
i z

R
f ( z ) e dz = 2 i

R
R

i z

CR
zk

f ( x ) sin xdx

k =1

Res f ( z ) ei z , zk

zk ( z = 1, 2,..., n ) : poles of f (z) in UHP

R
PV

f ( z )e dz =
i z

CR

real coefficients
P ( x)
, f ( x) =
, > 0 no real pole

f ( x ) ei x dx

Th. 19.16

Behavior of Integral as R

THEOREM 19.16

(Jordans Lemma)

Suppose f(z) = P(z)/Q(z), where the degree of P(z) is n and the degree of Q(z) is
m n + 1. If CR is a semicircular contour z = Rei, 0 , then
CR

f ( z ) ei z dz 0 as R

m > n is enough for non-integer m, n

189

Proof:

rId5

sin

y=

for

e R sin e 2 R /

R sin

d = 2

2
0

2
0

for R > 0 and 0


R sin

y = sin

Jordans inequality

(1 e )
R

d =

e R sin d <

i
Assume f(z): analytic at all points in the UHP that lie above a semicircle z = R0 e , 0

CR : z = Rei , R > R0 , 0
For all z on CR: f ( z ) M R
P( z)
=0
M R 0 as R lim
z Q ( z )
CR

f ( z ) ei z dz =

f ( Re
CR

) iRe

CR

m n +1

f ( Rei ) exp i Rei iRei d

MRR

exp i Re

f ( z ) ei z dz M R R

e R sin d <

e R sin d <

=e

R sin

M R R

R0

, R > 0
R

CR

f ( z ) ei z dz 0 as R
190

Th.

f ( z) =

P( z)

Q( z)

real coefficients
, P ( z ) , Q ( z ) :polynomial functions of z, deg Q ( z ) 1 + deg P ( z )

f ( z ) has no real poles (

Q ( z ) has no real zeros)

poles zk (k = 1, 2, , n) in UHP
PV

f ( x ) ei x dx = PV

f ( x ) cos xdx + iPV

Example 4

Evaluate the Cauchy principal value of

f ( x ) sin xdx = 2 i

Res ( fe ,3i ) = lim ( z 3i )


z 3i

PV

PV

- f(z) = P(z)/Q(z): real coefficient

- f(z) has no real poles

x
eix dx = PV
x 2 + 9

( z + 3i )( z 3i )

iz

x
cos xdx + iPV
x 2 + 9

x
i
sin xdx = 2 iRes ( feiz ,3i ) = 3
x +9
e

x sin x

dx = 3
2
x + 9
2e

x sin x
1
dx = PV
2
x +9
2

191

f ( x ) dx

CR

C = [ R, R ] +

f ( z ) dz +

rj 0

f ( x ) dx

z
eiz dz
z +9
2

f ( z ) has isolated singularities zku ( k = 1, 2,..., n ) in UHP


r
simple poles z j ( j = 1, 2,..., m ) on real axis

UHP

e 3
=
2

D. Indented Contours

PV

k =1

Res f ( z ) ei z , zk

x sin x
dx - deg[Q(x)] deg[zP(x)]
x2 + 9

iz

x sin x
dx even function
x 2 + 9
0
z
z
Simple poles: z1 = 3i, z2 = 3i
Let f ( z ) = z 2 + 9 = z + 3i z 3i
(
)(
)

Sol:

x sin x
1
dx =
2
x +9
2

m
j =1

( C ) + C
j

m
j =1

C j

CR

rj 0

iRes f ( z ) , z rj

Th.19.17

R
R

f ( z ) dz +

zku

f ( z ) dz =
R

f ( z )dz = 2 i
R

n
k =1

C j
z rj rj

Res f ( z ) , zku

rj 0

1. f (z) = P(z)/Q(z), deg Q(z) 2 + deg P(z)


2. f (z) = exp(iz) P(z)/Q(z), deg Q(z) 1 + deg P(z)

192

Behavior of Integral as r

THEOREM 19.17

Suppose f has a simple pole z = c on the real axis. If Cr is the contour defined
by z = c + rei , 0 , then
lim

r 0 Cr

f ( z )dz = iRes f ( z ) , c

Proof:
Laurent series
z = c : simple pole of f (z)

Cr

lim

r 0 Cr

z = c + rei , dz = irei d

g ( c + rei )ei d = I1 + I 2

id = ia1 = iRes f ( z ) , c

g ( z ) : analytic at z = c
I 2 = ir

a1
k
+ ak ( z c )
z c k =0

= g ( z ) analytic at z = c

a
f ( z ) = 1 + g ( z )
z c
i
ire
f ( z )dz = a1
d + ir
0 rei

I1 = a1

f ( z) =

g ( z ) M on Cr

g ( z ) : continuous at z = c

g ( c + rei )ei d r

M d = rM

f ( z )dz = iRes f ( z ) , c

lim
r 0

r 0

Cr

i.e., g(z) is bounded around c

f ( z )dz = iRes f ( z ) , c
193

Th.
f ( z ) has isolated singularities zku ( k = 1, 2,..., n ) in UHP
r
simple poles z j ( j = 1, 2,..., m ) on real axis

lim

R CR

f ( z )dz = 0

CR : z = Rei , 0
PV

f ( x ) dx = 2 i

n
k =1

Res f ( z ) , zku +

1
2

n
k =1

Res f ( z ) , z rj

1. f (z) = P(z)/Q(z), deg Q(z) 2 + deg P(z)


2. f (z) = exp(iz) P(z)/Q(z), deg Q(z) 1 + deg P(z)

194

Example 5
Evaluate the Cauchy principal value of
Sol:

Consider the integral


f ( z) =

sin x
dx

x ( x 2x + 2)

eiz
dz
z ( z 2 2z + 2)
Real axis

1
has simple pole: z r = 0 , z u = 1 + i, z l = 1 i
z ( z 2z + 2)
2

Res f ( z ) eiz , z u = lim

z (1+ i )

z (1 + i )

eiz

z z (1 + i )

UHP

z (1 i )

e 1
( sin1 cos1) i ( sin1 + cos1)
4
eiz
1
=
Res f ( z ) eiz , z r = lim z
2
z 0
z ( z 2z + 2) 2
ix

e
cos x
=
PV
dx
PV
dx + iPV
2
2

x ( x 2x + 2)
x ( x 2x + 2)

e 1+i
(1 + i )
4

lim

R CR

f ( z )ei z dz = 0

z = Rei , 0

Th. 19.16

sin x
dx
x ( x 2x + 2)
2

1
= 2 i Res f ( z ) eiz , z u + Res f ( z ) eiz , z r
2
=
PV

{e
2

( sin1 + cos1) + i

1 + e 1 ( sin1 cos1)

sin x

dx = 1 + e 1 ( sin1 cos1)

2
x ( x 2x + 2)

Ex:

(sin x / x) dx = /2

195

E. Integrals Involving Multiple-Valued Function


C

z a 1 f ( z )dz

z a 1 = e

z = rei
( a 1)
( a 1) ln z

=e

log e r + i ( + 2 n )

= r a 1ei( a 1)( + 2 n ) : multi-valued function

Analytic branch n = 0:

z a 1 = ( rei )

( a 1)

CR

: single-valued function

r > 0, 0 < < 2


z p = ei

Example

x a 1

, 0 < a <1
dx =
1+ x
sin a

Sol: Simple pole: z p = 1 = ei

R
z a 1
z a 1
z a 1
+ + +
, z p = ei
dz =
dz = 2 iRes

C 1+ z
CR
R
C
1+ z
1+ z
a 1
z a 1 i
z a 1
Res
, e = limi ( z ei )
= ( ei ) = ei a
z e
1+ z
1+ z

196

i
R
2 ( Re )
z a 1
0
dz =
Rei id

i
0
CR 1 + z
1 + Re
0 < a <1
z = Rei , dz = iRei d
a 1

0 ( e )
z a 1
dz =
ei id
2 1 + ei
C 1 + z
z = ei , dz = i ei d
a 1

z a 1
dz =
R 1+ z

i 2

( re )
i 2

1 + re

i 2

a 1

i 2

ei 2 dr = ei 2 a

CR

r a 1
dr
1+ r

branch
point
z p = ei

z = re , dz = e dr
a 1
a 1
R z
R r
dz =
dr
1+ z
1+ r
+

branch
cut

z = rei 0 , dz = ei 0 dr

For 0, R
2 i ( ei a ) = 0 ei 2 a

r a 1
dr + 0 +
1+ r

r a 1
dr
1+ r

r a 1
2 iei a
2 i

dr =
= i a i a =
i 2 a
1+ r
1 e
e e
sin a
197

Exercise
x a 1
dx = cot a , 0 < a < 1
0 1 x

(1 a )
xa
dx =
, 1 < a < 3
0
2 2
a
(1 + x )
4 cos
2

PV

xa
sin a
dx =
, a < 1, <
2
1 + 2 x cos + x
sin a sin

log e x
log e a
dx =
2
2
x +a
2a

( ln x )

1 + x2

dx =

ex.

[ln(1+x2)]/(1+x2) dx = ln2

2 2
ln x
=
dx
x4 + 1
16

( ln x )

3 3 2
dx =
x4 + 1
64
198

Exercise

cosh ax
dx =
cosh x

2 cos

, a <1

e az
dz
C cosh z

R +i

R + i

2
R

sin ax
1
a
1
dx = coth

2 x
e 1
4
2
2a

R+i

eiaz
dz
C e 2 z 1

199

Homework 7
19.5
20, 24, 30

19.6
18, 32, 34, 36

200

Contents
19.1 Sequences and Series
19.2 Taylor Series
19.3 Laurent Series
19.4 Zeros and Poles
19.5 Residues and Residue Theorem
19.6 Evaluation of Real Integrals
19.7 Summation of Series by the Residue Theorem
19.8 Inverse of Laplace Transforms

201

Th.
P ( z ) : polynomial of degree 2, with real coefficients and has no integer zeros
1
y
z j ( j = 1, 2, ..., m ) : zeros of P ( z ) integers
1
N + ( 1 + i )
N + (1 + i )
2
2

m
1
cot z
= Res
, zj
P( z)
n = P ( n )
j =1

( 1) = m Res
n = P ( n )
j =1
n

zj

csc z
, zj
P(z)

N 1

Proof:
Choose CN large enough st. all zj are inside CN
P ( z ) has zeros at z j ( j = 1, 2, ..., m ) integers
cos z
cot z =
has simple poles at n ( = 0, 1, 2,...)
sin z
f ( z) =

cos z
P( z)

g ( z)
sin z
P( z)
h( z)
has simple poles at n = 0, 1, 2, ...

cot z

N +1
x

CN
N+

1
2

( 1 i )

N+

1
(1 i )
2

poles at z j ( j = 1, 2, ..., m ) n

sin z = 0
zero of P ( z )
202

1
2 i

1
2 i

cot z
CN

P( z)

cot z
CN

P( z)

dz =

dz =

Res
N
n = N

cot z
P( z)

, zl

isolated singularities within CN

Res f ( z ) , n +

m
j =1

Res f ( z ) , z j

Since the degree of P(z) is k (k 2)


1
M
k , M : constant
P(z)
z

cot z
CN

lim

P( z)

dz A

cot z
CN

P( z)

dz = 0

N+

1
2

( 1 + i )

N+

1
M
k on CN
P( z) N

N
M
8N + 4)
k (
N
k2
cot z A for z CN

g ( n ) cos z / P ( z )
Res f ( z ) , n =
=
h '( n)
cos z

z =n

1
=
P (n)

1
(1 + i )
2

zj
N 1

N +1
x

CN
N+

m
1
cot z
= Res
, zj
P( z)
n = P ( n )
j =1

1
2

( 1 i )

N+

1
(1 i )
2

203

z1 + z2 z1 + z2
1
z
=
x
+
iy
, consider
z1 z2 z1 z2
2
i x y
i x + y
cos ( x + iy ) e
+e
cot z =
= i x y i x + y
sin ( x + iy )
e
e

N+

For y >

ei x y + e i x + y

1
2

( 1 + i )

N+

1
(1 + i )
2

zj

N 1

e y + e y 1 + e 2 y 1 + e
i x + y
=
=

A1
y
y
1 e 2 y 1 e
ei x y e e
e
1
1
y > e 2 y < e
For y < , consider z = x + iy
2
2

N +1
x

CN
1

N + ( 1 i )
e y + e y 1 + e 2 y 1 + e
2
cot z y y =

A1
2 y

e e
1 e
1 e
1
y < e 2 y < e
1
1
1
2
For y , consider z = N + + iy
2
2
2
1

cot z = cot N + + iy = cot


+ i y = tanh y tanh A2
2
2
2
1
consider z = N + iy
2
1

cot z = cot N + iy = tanh y tanh A2


2
2
A : constant
cot z < A, z CN
A > A1 , A > A2

N+

1
(1 i )
2

204

Example

1
, a : any positive constant
2
n =0 n + a
2

- deg[P(z)] 2
- P(z): real coefficients
- P(z) has not integer zeros

P ( z ) = z 2 + a 2 : polynomial of degree 2, zeros at ai

1
1
1 n=0
2
=

2
2
2
2
a2
n = n + a
n=0 n + a

1
=
n = P ( n )

cot z
Res 2
, zj
z + a2

1
=
2
2
n = n + a

= Res

Res
j =1

cot z
, zj
P( z)

cot z
cot z
, ai + Res 2
, ai
2
2
z +a
z + a2

cot ai cot ai
coth a
+
=
2ai
2ai
a

1
coth a 1
=
+ 2
2
2a
2a
n=0 n + a

205

Example

( 1)

n=0

( 2n + 1)

3
32

n=0

n = 1

n=1

1/33

n = 2

1/33

n=2

1/53

n = 3

1/53

n=3

1/73

n = 4

1/73

3
P ( z ) = ( 2 z + 1) : polynomial of degree 3, zeros of order 3 at

( 1)

n =

( 2n + 1)
( 1)

n =

n
3

=2

n =0

( 1)

( 2n + 1)

( 2n + 1)

n
3

csc z

Res

1
d2
lim 2
2! z 1 dz

1
2

( 2 z + 1)

z+

, z j = Res
1
2

csc z

( 2 z + 1)

1
2

csc z

( 2 z + 1)

d2
3

lim 2 [ csc z ] =
16 z 1 dz
16
2

n=0

( 1)

( 2n + 1)

32

206

Exercise

n=0

n =

( 2n

1
2

+ 1)

( 1)

(a + n)

1 2

csch
+ 2 cosh
+4
8
2
2

= 2 csc ( a ) cot ( a )

a : positive number
a integer

207

Contents
19.1 Sequences and Series
19.2 Taylor Series
19.3 Laurent Series
19.4 Zeros and Poles
19.5 Residues and Residue Theorem
19.6 Evaluation of Real Integrals
19.7 Summation of Series by the Residue Theorem
19.8 Inverse of Laplace Transforms

208

Laplace Transform and Inverse Laplace Transform


Def.
f ( t ) , t [ 0, )

Laplace transform of f : L f ( t ) =
converges

f ( t )e st dt = F ( s ) , provided the integral

Inverse Laplace transform of f : u ( t ) f ( t ) =L 1 F ( s )


u (t ) =

1, t > 0
0, t < 0

Example
f ( t ) = e zt , z : complex, t > 0

L f (t ) =
=

e zt e st dt =

( s z )t

e
( s z)

sz t
( )
0

L 1 F ( s ) = e zt , t > 0

dt

1
= F ( s)
sz

Re ( s z ) > 0, Re s > Re z

, exp((sz)t) = 0

as t

209

Th.
F ( z ) =L f ( t ) =

f ( t )e zt dt

a + iR

F ( z ) analytic in D1 = { z Re z > a}
lim F ( z ) = 0, z D1
z

or |zF(z)| < M as z

u ( t ) f ( t ) = L 1 F ( z ) =

Proof:

F (s) =

1
2 i

1
2 i

a + i
a i

La

F ( z )e zt dz

s D1 = { z Re z > a}

C1 = La + CR1

CIF 2 iF ( s ) =

CR1

F ( z)
dz =
C1 z s
a + i
a i

a + iR
a iR

F ( z)
dz
sz

D1

a iR

u ( t ) f ( t ) =L 1 F ( s ) =L 1
u ( t ) f ( t ) = L 1 F ( z ) =

F ( z)
F ( z)
dz +
dz
C
R1 z s
zs
0, lim F ( z ) = 0, z D1

1
2 i

1
2 i

a + i
a i

a + i
a i

1
F ( z)
dz =
2 i
sz

a + i
a i

1
F ( z ) L 1
sz

dz

F ( z ) e zt dz
210

Th.
F ( z ) =L f ( t ) =

f ( t )e zt dt

F ( z ) analytic over entire complex plane except at a finite number of poles

( j = 1, 2,..., n ) in D2 = { z Re z < a}
F ( z ) analytic in D1 = { z Re z > a}
zF ( z ) M , z R

zj

u ( t ) f ( t ) = L 1 F ( z ) =

j =1

a + iR

CR 2

Res F ( z ) e zt , z j

La

zj

Proof:
u ( t ) f ( t ) = L 1 F ( z ) =
1
2 i

= lim

1
2 i
n
j =1

C2

1
2 i
a + iR
a iR

a + i
a i

F ( z ) e zt dz +

F ( z )e zt dz

CR 2

F ( z )e zt dz
F ( z ) e zt dz

C2 = La + CR 2

) R

Jordans lemma

D2

a iR

Res F ( z ) e zt , z j
211

Example

check deg P and Q first!!

1
,b > 0
z + b2
e zt
e zt
ib
Res 2
,

=
z + b2
2z
F ( z) =

simple poles at z = ib

z = ib

ib

e ibt
i 2b

a>0

ib

e zt
eibt e ibt sin bt
Res 2 2 , z j =

=
z +b
i 2b i 2b
b

u (t ) f (t ) =

Example
F ( z) =

Res

Res

(z

(z
(z

4 ) ( z 1)
e zt

4 ) ( z 1)
e zt

4 ) ( z 1)

simple poles at z = 2
pole of order 2 at z = 1

, 2 = lim ( z 2 )
2
z 2

(z

, 2 = lim ( z + 2 )
2
z 2

e zt

4 ) ( z 1)

(z

4 ) ( z 1)

a>2

1
= e 2t
4

e zt

1 2t
e
36
212

Res

(z

e zt

4 ) ( z 1)

u (t ) f (t ) =

d
z 1 dz

,1 = lim
2

( z 1)

(z

e zt

4 ) ( z 1)

1
2
= te t et
3
9

e zt

1
1
1
2
, z j = e2t e2t tet et
4
36
3
9
( z 4) ( z 1)

Res

Exercise
L

L 1

( z 2) ( z + 4)
2

cos 2 z

( z 2)

1
( 6te2t e2t + e4t )
36

1
= e 2t ( t 2 + t 2 cos 4 4t sin 4 4 cos 4 )
4

L -1 (1/ s + 1) = 1/2i

a + i

exp(zt)/ z + 1 dz = exp(-t)/ t

a i

213

Chapter 20
Conformal Mappings and Applications

214

Contents
20.1 Complex Functions as Mappings
20.2 Conformal Mapping and the Dirichlet Problem
20.3 Linear Fractional Transformations
20.4 Schwarz-Christoffel Transformations

215

Mappings Between Planes


w = f ( z ) = u ( x, y ) + iv ( x, y )

planar transformation (mapping)

w = f (z)
R

w
C'

R'
u

z-plane
z = x + iy
C : z ( t ) = x ( t ) + iy ( t ) , a t b

R = { z}

w-plane
w = u + iv = f ( z )
C ' : w (t ) = f z (t )

R ' = w w = f ( z), z R

216

Example 1

w = f ( z ) = e z, R = { z = x + iy 0 y }, R ' = ?

Sol:
- Horizontal line on z-plane
y=b

y
y =

z ( t ) = t + ib, < t < , 0 b

y=b

w = f ( z ) = e z = et +ib = et eib

y=0

L3

L4

x=a

L2

L1

w = et , Arg w = b
y=0

z ( t ) = t + i0

w = et + i 0 = et = u > 0

y =

z ( t ) = t + i

w = et +i = et = u < 0

v
R'

L4 ' L2 '

- Vertical line
x=a

Arg w = b

z ( t ) = a + it , < a < , 0 t

w = ea +it = ea eit

L3 '

w = e , Arg w = t
a

L1 '

217

R = {z 0 y }

R ' = w = e z 0 Arg w , w 0

upper half plane


v

y
R'

y =
L3

R
y=b

L4 x = a

L2

y=0

w = e = f ( z)

Arg w = b

z = Ln w = f 1 ( w )

ea

L1

L3 '

Example 2
w = f ( z) =

1
1
x
y
=
= 2
i 2
= u ( x, y ) + iv ( x, y )
2
z x + iy x + y
x + y2

- Level curve
u ( x, y ) =

L1 '

w-plane

z-plane

Sol:

L4 ' L2 '

x
= a ( 0)
2
x + y2

1
2a

+ y2 =

A circle with center at

1
2a
1
1
, 0 , radius of
2a
2a

218

A circle with center at

- Level curve
v ( x, y ) =
y

y
= b ( 0)
2
x + y2

1
x + y+
2b
2

0,

1
=
2b

1
1
, radius of
2b
2b

1
2a

C1

w=
x

1
z

v=b

C2 '

C1 '
u=a

1
z=
w

1
2b
C2

w-plane

z-plane

y=b

1
2a

C4

C3
x=a
x

1
= f ( z)
z
1
z = = f 1 ( w )
w
C3 '

w=

f = f 1

w=

1
z

z=

1
w

1
2b

z-plane

C4 '

219

w-plane

Translation and Rotation


Translation in z-plane
w = f ( z ) = z + z0 : translated or shifted by z0
w = z + z0 = ( x + h ) + i ( y + k )

w = z + z0

= ( x + h) + i ( y + k )

x + iy

x
z -plane

w = z + z0

y+k

x
w-plane

x+h

220

Rotation through 0 degree

( )

w = g ( z ) = ei0 z : rotated by 0 degree

( )

w = ei0 z = ei0 rei = re (

i +0 )

( )z

w= e

z = rei
x

i0

w = re (

i +0 )

w-plane

z -plane

221

Translation and rotation

( )

w = h ( z ) = ei0 z + z0

= ( ei ) z rotated by 0
0

w = + z0

shifted by z0

( )

w = + z0 = ei0 z + z0

w = + z0

= ( ei ) z
0

z -plane

R'

z0

R ''

-plane

w-plane

222

Example 3
Find a complex function that maps the horizontal strip 1 y 1 onto the
vertical strip 2 x 4
Sol:

w = +3

z -plane
y

= iz + 3

R = { z = x + iy 1 y 1}

w-plane

w = f ( z)

= iz + 3

R'

R ' = {w = u + iv 2 u 4}

-plane

= ( ei 90 ) z = iz

w = +3

R ''

Rotated by 90
1

Shifted by 3

223

Magnification
w = f ( z ) = z : magnified by factor

: fixed positive real number


w = z
i
w = g ( z ) = az + b , a = r0 e 0

= az = ( r0 ei ) z
0

w = +b

rotated by 0, magnified by r0

shifted by b

Example 4
Find a complex function that maps the disk |z| 1 onto the disk
|w (1 + i)| 1/2

224

Sol:

z 1

w (1 + i )

w = + (1 + i )

=
x

z
+ (1 + i )
2

R'

1+ i

z -plane

1
2

w-plane

w = + (1 + i )

z
2

Shifted by (1 + i)

R ''

Magnified by 1/2

-plane

225

Real Power Functions


w = f ( z ) = z

w = z = ( re

= fixed positive real number

=r e

Angle expanded

Amplitude magnified

r
v

Angular wedge

R = { z = re 0 Arg z = 0 }
i

w = z = r ei

C'

R ' = {w = ei 0 Arg w = 0 }

226

Example 5
Find a complex function that maps upper half-plane y 0 onto the wedge
0 Arg w /4
Sol:

0 =

0 = 0 =

Angle expanded by 1/4


y

w = z = z 4

0 =
z -plane

0 =

w-plane

R'

227

Successive Mappings
= f ( z)
w = g ( )

Note : F can also be denoted as g o f

w = g ( ) = g f ( z ) = F ( z )
To find w = F(z)

228

Example 6
Find a complex function that maps horizontal strip 0 y onto the wedge
0 Arg w /4
Sol:
y

v
1

w = 4 = e4

R'

z -plane

=e

w-plane

w = 1/4

R ''

-plane

R '' = { 0 Arg }

229

Example 7
Sol:

Find a complex function that maps the wedge /4 Arg z 3/4 onto the upper
half-plane v 0
y

w = iz 2
R'

z -plane

= e

w-plane

R ''

w =2

-plane

w= =
2

= iz 2

230

Contents
20.1 Complex Functions as Mappings
20.2 Conformal Mapping and the Dirichlet Problem
20.3 Linear Fractional Transformations
20.4 Schwarz-Christoffel Transformations

231

Angle-Preserving Mappings

Note : z1, z2, w1, w2 : tangent vectors

Complex mapping w = f ( z ) defined in domain D


z-plane

z0

w-plane

w0

Def.
Mapping w = f ( z ) conformal at z0 in D
=

f preserves both magnitude and sense of angle


232

THEOREM 20.1

Conformal Mapping

If f is analytic in domain D and f ' ( z0 ) 0, then f is conformal at z = z0


Proof: (1)
w = f z ( t ) : image curve in w-plane

Curve C in D: z = z ( t )
w ' = f ' z (t ) z ' (t )

C1 : z1 ( t ) w1 = f z1 ( t )

If two curves C1 and C2 intersect in D at z0


w1 ' = f ' ( z0 ) z1 '

w2 ' = f ' ( z0 ) z2 '

f ' ( z0 ) z1 ' + f ' ( z0 ) z2 ' f ' ( z0 ) z1 ' f ' ( z0 ) z2 '


2

= cos

= cos

f ' ( z0 ) 0

C2 : z2 ( t ) w2 = f z2 ( t )

2 f ' ( z0 ) z1 ' f ' ( z0 ) z2 '

z1 ' + z2 ' z1 ' z2 '


2 z1 ' z2 '

z1 ' z2 ' = z1 ' + z2 ' 2 z1 ' z2 ' cos

233

(2)
C : z = z ( t ) : curve in D through z0

C ' : w ( t ) = f z ( t ) : curve in w-plane through w0 = f ( z0 )

z: any point in D
w w0 =

f ( z ) f ( z0 )
( z z0 )
z z0

arg ( w w0 ) = arg

= arg w ( t ) w0

f ( z ) f ( z0 )
+ arg ( z z0 )
z z0

z = z ( t ) C z0 = z ( t 0 )
z z0
= arg z ( t ) z0

w = f z (t ) C '

w-plane

z-plane

w0 = f z ( t0 ) = f ( z0 )

z = z (t )

w = f z (t )
C'

z0

w0

w0

z0

arg ( z z0 )

arg ( w w0 )

234

z z0

f ( z ) f ( z0 )
z z0

arg ( w w0 ) = arg

f ' ( z0 ) 0

f ( z ) f ( z0 )

= arg f ' ( z0 )

1. Since f is analytic f (z0) exists.


2. If f (z0) = 0, then arg [f (z0) ] is undefined,
and z0 is a critical point in the mapping.

z z0

+ arg ( z z0 )

z z0

= arg f ' ( z0 ) +

: independent of curve

Let C1 and C2 be intersected at z0


C1 ' = f ( C1 )
C2 ' = f ( C2 )

intersect at w0 = f ( z0 )

C2 '

C1 '

C2

1 1 = arg f ' ( z0 )

z0

2 2 = arg f ' ( z0 )

w0

C1

1 1 = 2 2

w-plane

z -plane

= 2 1 = 2 1 =

w = f ( z ) conformal at z0
235

Example 1
f ( z ) = e z Ch. 20 p. 217
f '( z ) = ez 0

f ( z ) = e z is conformal at all points in the z-plane

g ( z ) = z 2 Ch. 17 p. 24
g ' ( z ) = 2 z 0 for z 0
g ' ( z ) = 2 z = 0 at z = 0

g ( z ) = z 2 is conformal at all points except at z = 0


y

g '' ( z ) = 2 0

Note:

g ' ( z0 ) = 0, g '' ( z0 ) 0

= 2

z -plane

Note: If g(k) (z0) = 0 for k = 1 n1, but g(n) (z0) 0


= n

Example 2
w = f ( z ) = sin z , R = z = x + iy

w-plane

: fundamental region of sin z

R' = ?
236

Sol:
Vertical line: x = a, z ( t ) = a + it ,

, < t <
2
2
w = u + iv = sin ( a + it ) = sin a cosh t + i cos a sinh t

u ( t ) = sin a cosh t

u
sin a

v ( t ) = cos a sinh t

cos a

= 1 hyperbola

v = 0, u = sin a, u < 1
x=a=
x=a=

v = 0, u = cosh t : ( , 1]
v = 0, u = cosh t : [1, )

Horizontal line: y = b, z ( t ) = t + ib,

2
2
w = u + iv = sin ( t + ib ) = sin t cosh b + i cos t sinh b

u ( t ) = sin t cosh b
v ( t ) = cos t sinh b

u
cosh b

v
+
sinh b

, < b <

=1

ellipse

w = u = sin t : [ 1,1]

y = b = 0, z = t + i 0

v=0

w = sin z : R = z = x + iy

237

R ' = w = u + iv

w = sin z conformal mapping at all points in R except z =


f ' ( z ) = cos z = 0 for z =

f '' ( z ) = sin z 0 for z =

hyperbola ellipse

z=

, = 180

w = 1, = 2 = 360

u
= whole w-plane
v

2
BAD = CAD = 90
B ' A ' D ' = C ' A ' D ' = 180

BAC = 180
B ' A ' C ' = 360

238

Example 3

1
w = f ( z ) = z + analytic at all points except z = 0
z
1 z2 1
f ' ( z ) = 1 2 = 2 = 0 for z = 1
z
z
1
w = f ( z ) = z + conformal at all points except z = 0, 1
z

R' = ?

R = z z 1, Im z 0
z = rei

Sol:

1
1
1
= rei + e i = r ( cos + i sin ) + ( cos i sin )
z
r
r
1
1
= r + cos + i r sin = u + iv
r
r

w = f ( z) = z +

u= r+

1
1
cos , v = r sin
r
r
v = 0, u = 2 cos

r=1

semicircle z = eit , 0 t

segment [2, 2]

239

u= r+

r > 1, semicircle z = re , 0 t
it

u
r + (1/ r )

v
+
r (1/ r )

u
a

1
1
cos t , v = r sin t
r
r
v
b

a = r + (1/ r )

=1

b = r (1/ r )

Ellipse in UHP
v0

is fixed, ray z = tei , t 1 u = t + t cos , v = t t sin


u
cos

sin

1
= t+
t

1
t
t

=4

ellipses hyperbolas

f conformal in R

Hyperbola in UHP
v0
Note: for z = 1,
f (z) = 0, but f (z) 0
DAB = 90
D ' A ' B ' = 180

R = z z 1, Im z 0

R' = ?
240

Conformal Mapping Using Table (Appendix IV)


Example 4

R ' = {w = u + iv v 0}

w = f ( z ) : R = { z = x + iy 0 y 2}

Sol:
v

y
C

ai

H-2 z

w=ea

1
F'

B' C' D' E'

A'

w = f (z) = e 2

a=2

241

Example 5

w = f ( z ) : R = { z = x + iy 0 y 2}

Sol:
z

i 1 e a
w=
=
z
i +
1+ e a

ai

E'

D' u

R ' = w = u + iv w 1
v

H-2 : = e a

C-4 : w =

1
D '' E ''
z

a=2

w = f ( z) =

ie 2

i
i +

bilinear transformation, or
linear fractional transformation

i+e 2

242

Harmonic Functions and Dirichlet Problems


Dirichlet Problem:
2
2
+
P ( x, y ) = 0, ( x, y ) R
x 2 y 2

D.E.:

B.C.: P ( ) = V0 : given

R = region in ( x, y ) -plane
: boundary of R

Neumann Problem:
2
2
+
P ( x, y ) = 0, ( x, y ) R
x 2 y 2
P
B.C.:
( ) = V0 : given
n

D.E.:

Mixed Type Problem:


2
2
+
P ( x, y ) = 0, ( x, y ) R
x 2 y 2
P
B.C.: aP() + b ( ) = V0 : given
n

D.E.:

243

THEOREM 20.2

Transformation Theorem for Harmonic Functions

w = f ( z ) is analytic: D = { z = x + iy}
f '( z ) 0

D ' = {w = u + iv}

( x, y ) = ( f ( z ) ) = ( z ) harmonic in D
( u, v ) = ( w ) harmonic in D '

Proof:
2

+ 2 = f '( z )
+ 2
2
2
x
y
u
v
2

f '( z ) 0

2 2
+
=0
x 2 y 2

2 2
+
=0
u 2 v 2
: harmonic in D '

244

w = f ( z ) is analytic:D = { z = x + iy}

D ' = {w = u + iv}

2
2
2 2
2
+
=
+
f
'
z
(
)
x 2 y 2
u 2 v 2

Proof:
( x, y )

x ( u, v ) , y ( u, v )
u v
=
+
y u y v y

u v
=
+
x u x v x

chain rule

2 2u u
2v v
=
+
+
+
x 2
u x 2 x x u
v x 2 x x v

2u u u v
2 v v u v
+
+
+
+
+
u x 2 x u u x v u x
v x 2 x u v x v v x

2u u 2 u 2 v
2 v v 2 u 2 v
=
+
+
+
+
+
u x 2 x u 2 x vu x
v x 2 x uv x v 2 x
245

2 2u u 2 u 2 v
2 v v 2 u 2 v
=
+
+
+
+
+
x 2
u x 2 x u 2 x vu x
v x 2 x uv x v 2 x
2 2u u 2 u 2 v
2 v v 2 u 2 v
=
+
+
+
+
+
y 2
u y 2 y u 2 y vu y
v y 2 y uv y v 2 y
f '( z )
2 2 2u 2u
2 v 2 v
2
+
=
+
+
+
+
x 2 y 2
u x 2 y 2
v x 2 y 2
u 2
+2

2 u v u v
+
vu x x y y

w = f ( z ) = u + iv : analytic
u
x

u
+
y

v
=
x

v
+
y

u
x

u
=
x

v
+
x

2
+ 2
v

v
x

v
+
y

CR equation

u, v: harmonic

f '( z )

u
+
y

u v v
u
= ,
=
x y x
y

2u 2u
2v 2v
+
=
0,
+
=0
x 2 y 2
x 2 y 2
2

2
u v
=
+i
= f '( z )
x
x

u v u v
+
=0
x x y y
2
2
2 2
2
+
=
+
f
'
z
(
)
x 2 y 2
u 2 v 2

246

Solving Dirichlet Problems Using Conformal Mapping


Original Dirichlet problem:
D.E.:

2
2
( x, y ) = 0, ( x, y ) R
+
x 2 y 2

B.C.: ( 1 ) = V1
( 2 ) = V2

: given

= 1 + 2 : boundary of R

To find conformal mapping w = f ( z )


Complicated

w = f ( z ) : analytic
f '( z ) 0

R
1 , 2

Simpler
R'
1 ', 2 '

2
2
+
( u , v ) = 0, ( u , v ) R '
u 2 v 2

2
2
( x, y ) = 0, ( x, y ) R
+
x 2 y 2

Transformed Dirichlet problem


D.E.:

2
2
+
( u , v ) = 0, ( u , v ) R '
u 2 v 2

B.C.: ( 1 ') = V1
( 2 ') = V2

Same D.E.
Simpler B.C.

: same given values

247

=1

=1

=0

=2

( f ( ) ) = ( )

=0
=2

= 1

= 1

Uniqueness theorem for both Dirichlet and Neumann problems


Dirichlet problems: (x, y) satisfying D.E. in R and B.C. on is unique
Neumann problems: (x, y) satisfying D.E. in R and B.C. on differs at most by an
additive constant

248

Example 6
The function Q ( u, v ) = (1/ ) Arg w is harmonic in the upper half-plane v > 0
since it is the imaginary part of the analytic function g ( w ) = (1/ ) Ln w . Use
this function to solve the Dirichlet problem in the figure.
Sol:
q =1

q=0

q =1

D.E.:

q=0

Q =1 Q =1 Q = 0 Q = 0

2
2
+
q ( x, y ) = 0, ( x, y ) the interior of R
x 2 y 2

q ( ODE ) = 0

B.C.: q ( OAB ) = 1

w = f ( z ) = sin z : R = z = x + iy

, y0

BAODE

R ' = {w = u + iv v 0}

B ' A'O ' D ' E '

249

2
2
+
Q ( u , v ) = 0, ( u , v ) the interior of R '
D.E.:
u 2 v 2

Q ( O ' D ' E ') = 0


1
1
g ( w ) = P ( u, v ) + iQ ( u, v ) = Ln w = ( log e w + iArg w ) analytic in R '

B.C.: Q ( O ' A ' B ') = 1

Q ( u, v ) =

Arg w

: harmonic in R and satisfies the boundary conditions


Q(u,v) is the solution

q ( x, y ) = Q ( f ( z ) ) = Q ( sin z ) =
=
=

Arg ( sin z )

Arg ( sin x cosh y + i cos x sinh y )


tan 1

cos x sinh y
sin x cosh y

250

Example 7
From C-1 in Appendix IV of conformal mappings, the analytic function
f ( z ) = ( z a ) / ( az 1) , where a = 7 + 2 6 / 5, maps the region outside the two
open disks z < 1 and z ( 5 / 2 ) < 1/ 2 onto the circular region r0 w 1 ,
where r0 = 5 2 6 . The figure shows the original Dirichlet problem and the
transferred boundary conditions.
P=0

Sol:
p=0

'A

p =1

r = r0

P =1

'B
r =1

C-1 in Appendix IV: w = f ( z ) =

za
,
az 1

7+2 6
5

R ' = w = u + iv r0 w 1

A ' = w = u + iv w = 1

{
}
' = {w = u + iv w = r }

a=

251

2
2
+
P ( u , v ) = 0, ( u , v ) R '
D.E.:
u 2 v 2

P ( B ') = P ( r = r0 ) = 1

B.C.: P ( A ') = P ( r = 1) = 0
g ( w ) = P ( u, v ) + iQ ( u, v ) =
P ( u, v ) =

log e w
log e r0

log e r
: harmonic in R and satisfies the boundary conditions
log e r0

P(u,v) is the solution

p ( x, y ) = P ( f ( z ) ) =

1
1
Ln w =
( loge w + iArg w) analytic in R '
log e r0
log e r0

log e f ( z )

log e 5 2 6

log e r0

1
za
=
log e
az 1
log e r0

( x + iy )

log e

7+2 6
5

7+2 6
5

( x + iy ) 1

P=0

'A
r = r0

P =1

'B
r =1
252

C , a<u<b

0
Note: Dirichlet problem with B.C. of Q1 ( u, v ) = 0, otherwise

a, b: real numbers, a < b


R ' = {w = u + iv v 0} = upper half plane

(1) g ( w ) = P1 ( u, v ) + iQ1 ( u, v ) = Ln ( w a ) = log e w a + iArg ( w a ) analytic in R '


v

Q1 ( u , v ) = Arg ( w a ) harmonic in R '

w
R'

Q1 ( u > a, v = 0 ) = 0
Q1 ( u < a, v = 0 ) =

(2) Q2 ( u, v ) = Arg ( w b ) harmonic in R '


Q2 ( u > b, v = 0 ) = 0

(3) Q ( u, v ) =

Q = C0

Q=0

Arg ( w b )

Q=0

Q2 ( u < b, v = 0 ) =
C0

Arg ( w a )

Arg ( w b ) Arg ( w a ) harmonic in R '

Arg ( w a )

Q ( u > b, v = 0 ) = 0 0 = 0
Q ( u < a, v = 0 ) = C0 C0 = 0
Q ( a < u < b, v = 0 ) = C0 0 = C0

R'

a
Q1 =

Q1 = 0
253

Homework 8
20.1
4, 17, 25

20.2
6, 10, 15, 21, 25

254

Contents
20.1 Complex Functions as Mappings
20.2 Conformal Mapping and the Dirichlet Problem
20.3 Linear Fractional Transformations
20.4 Schwarz-Christoffel Transformations

255

Linear Fractional Transformation


a, b, c, d : complex constants with ad - bc 0
az + b
: Linear fractional transformation
cz + d
ad bc

w = T ( z) =
T '( z ) =

( cz + d )

w = T ( z ) conformal at z , if = ad bc 0 and z

Note: c 0
T(z) has simple pole at z0 =

d
c

if ad bc = 0

T '( z ) = 0

d
c

b
z =a
T ( ) = lim T ( z ) = lim
z
z
d c
c+
z
a+

Example 1

If T ( z ) = ( 2 z + 1) / ( z i ), compute T ( 0 ) , T ( ) , T ( i )

Sol:

1
T ( 0) = = i
i

1
z =2
T ( ) = lim
z
i
1
z
2+

T ( i ) = lim T ( z ) =
z i

256

Circle-Preserving Property
c=0
w = T (z) =

= Az

az + b
= Az + B, linear function
d

rotation-magnification

w = + B translation

c0
w = T (z) =

az + b
bc ad
=
cz + d
c

Let A = bc ad

B=

w = Az2 + B

a
c

1
a
+
cz + d
c
z2 =

z1 = cz + d

1
z1

inversion

translation + rotation-magnification

Linear fractional transformation = translation + rotation-magnification +


inversion
circle
circle
Translation or rotation-magnification :
straight line
straight line
257

1
z
2
2
K : (x + y ) + x + y + = 0

Inversion w =

, , , : real

0
straight line, = 0
circle,

z = x + iy, z = x iy, z = x 2 + y 2 , z + z = 2 x, z z = i 2 y
2

K: z +
z = 1/ w

K ':

1
1 1
1 1
+
+
+

+ = 0
w
2 w w
2i w w
2

K ': w +
w =

z+z
zz
+
+ = 0
2
2i

w+ w
w w

+ = 0
2
2i

circle,
0
straight line, = 0

1
:
z

circle or straight line K

circle or straight line K

K Circle or straight line passes through z = 0 (pole)


K straight line

=0

=0

K ' straight line

K ' circle or straight line passes through w = 0

Pole of T 1 ( w ) = z = 1/ w in w-plane
258

THEOREM 20.3

Circle-Line-Preserving Property

A linear fractional transformation maps a circle in z-plane to either a line or a


circle in the w-plane. The image is a line if and only if the original circle passes
through a pole of the linear fractional transformation.
Note:

Note:

az + b
cz + d
2. K circle or straight line in z -plane

K circle or straight line passing through pole z0

1. w = T ( z ) =

of T ( z )

K ' straight line

3. K ' = T ( K ) : image of K under T

K straight line

K ' circle or straight line in w-plane

K ' circle or straight line passing through


dw b
the pole of T 1 ( w) = z =
in w-plane
cw + a

259

Example 2
Find the images of the circles |z| = 1 and |z| =2 under T(z) = (z + 2)/(z 1).
What are the images of the interiors of these circles?
1 '

Sol:

R1 '

z2

1
z1

z3
R1

z0

w3

w1

w2

w = T ( z) =

z+2
z 1

(1) 1 = { z z = 1}: passes through pole z0 = 1 of T(z)


z1 = 1
z2 = i
z3 = 0

1 + 2
1
=
1 1
2
i + 2 1 3
= i
w2 =
i 1 2 2
0+2
w3 =
= 2
0 1
w1 =

R1 = z z < 1

1 ' = line

1 ' = w = u + iv u =

R1 ' = w = u + iv u <

1
2

1
2

260

z4

z5

z0

z3

2 '

R2 '

w3

w4

w5
2

R2

(2) 2 = { z z = 2} : does not pass through pole z0 = 1


z =2

T ( z) =

z =2

T ( z) = w

T ( z ) = T ( z) = w

z4 = 2

w4 = 0

z5 = 2

w5 = 4

2 = z z = 2
z3 = 0

2 ' = circle

z+2
z +2
=
=T (z )
z 1
z 1

Image circle 2 ' is symmetric w.r.t. u-axis


Center of the circle: w = 2, radius = 2

2 ' = w w 2 = 2
w3 = 2

R2 = z z < 2

261

R2 ' = w w 2 > 2

Constructing Special Mappings


w = T ( z) =

az + b
: circle or line
cz + d

To find w = T ( z ) =

az + b
: z1 , z2 , z3
cz + d

Then w = T ( z ) :

'

R'

' circle or line


w1 , w2 , w3 '

'

Note: Whether the image is upper or


lower half plane needs verification

262

Matrix Method
az + b
T ( z) =
cz + d

= T1 ( z ) =

a1 z + b1
c1 z + d1

T1 =

a1 b1
c1 d1

w = T2 ( ) =

a2 + b2
c2 + d 2

T2 =

a2
c2

then w = T2 (T1 ( z ) ) = T2 ( ) =

a2 + b2
=
c2 + d 2

a1 z + b1
+ b2
c1 z + d1

c2

a1 z + b1
+ d2
c1 z + d1

T=

az + b
cz + d

T=

z = T 1 ( w ) =

Example 3
If T ( z ) =

b2
d2

a2

T ( z ) = T2 (T1 ( z ) )
w = T ( z) =

Sol:

Note: T is not unique; only 3


elements are independent

a b
T=
c d

a b
c d

( a1a2 + b2c1 ) z + ( a2b1 + b2 d1 )


( a1c2 + c1d2 ) z + ( b1c2 + d1d 2 )
a ( a z + b1 ) + b2 ( c1 z + d1 )
az + b
= 2 1
= T ( z) =
c2 ( a1 z + b1 ) + d 2 ( c1 z + d1 )
cz + d

a2

b2

a1

b1

c2

d2

c1

d1

a1a2 + b2 c1

a2b1 + b2 d1

a1c2 + c1d 2

b1c2 + d1d 2

= T2 T1

a b
c d
d b
= adj T
c a

dw b
cw + a

263

z i
2z 1
and S ( z ) =
, find S 1 (T ( z ) )
iz 1
z+2

T ( z) =

2z 1
z+2

T=

S ( z) =

z i
iz 1

S=

S 1 (T ( z ) ) =

2 1
1

1 i
i 1

az + b
cz + d

a b
1 i
= ( adj S ) T = adj
c d
i 1
S 1 (T ( z ) ) =

2 1
1 i 2 1
2 + i 1 + 2i
=
=
i 1 1 2
1 2
1 2i 2 + i

( 2 + i ) z + (1 + 2i )
(1 2i ) z + ( 2 + i )

264

Triples to Triples
T ( z) =

( z z1 )( z2 z3 )
( z z3 )( z2 z1 )

: an alternative representation of T ( z ) =

T ( z1 ) = 0

zero at z1

T ( z3 ) =

pole at z3

T ( z2 ) = 1

( z z1 )( z2 z3 ) : z , z , z
1
2
3
( z z3 )( z2 z1 )
( z z1 )( z2 z3 ) : cross ratio of z , z , z
1
2
3
( z z3 )( z2 z1 )

w = T ( z) =

S ( w) =

( w w1 )( w2 w3 ) w , w , w
: 1 2 3
( w w3 )( w2 w1 )

S 1 : 0, 1,

az + b
cz + d

w1 = 0, w2 = 1, w3 =

0, 1,

w1 , w2 , w3

265

= T ( z ) : z1 , z2 , z3

1 = 0, 2 = 1, 3 =

w = S 1 ( ) : 1 = 0, 2 = 1, 3 =

w1 , w2 , w3

w = S 1 ( ) = S 1 (T ( z ) ) = f ( z ) : z1 , z2 , z3
w = S 1 (T ( z ) )

S ( w) = T ( z )

w = f ( z ) = S 1 (T ( z ) ): z1 , z2 , z3
w = f ( z)

z1 , z2 , z3

= T ( z)

w1 , w2 , w3

( w w1 )( w2 w3 ) = ( z z1 )( z2 z3 )
( w w3 )( w2 w1 ) ( z z3 )( z2 z1 )
0, 1,

w1 , w2 , w3

w=S

( )

w1 , w2 , w3

266

Example 4
Construct a linear fractional transformation that maps the points 1, i, and 1 on
the circle |z| = 1 to the point 1, 0, and 1 on the real axis.
Sol:

( w ( 1) ) ( 0 1) = ( z 1) ( i ( 1) )
( w 1) ( 0 ( 1) ) ( z ( 1) ) ( i 1)

w +1
z 1
=i
w 1
z +1

w = T ( z ) = i

z i
z +i

Example 5 Note: If zk = , replace each factor containing zk by 1 in the cross ratio


Construct a linear fractional transformation that maps the points , 0, and 1 on
the real axis to the point 1, i, and 1 on the circle |w| =1.
Sol:

( w 1) ( i ( 1) ) ( z )( 0 1)
=
( w ( 1) ) ( i 1) ( z 1)( 0 )
S ( w) =
S=

( w w1 )( w2 w3 ) = ( z z1 )( z2 z3 )
( w w3 )( w2 w1 ) ( z z3 )( z2 z1 )

iw + i 0 1
=
= T ( z)
w +1
z 1

i i
1 1

S 1T

w = S 1 (T ( z ) )

adj S =

S 1

1 i
1 i

1 i
1 i

0 1
i 1 + i
=
1 1
i 1 + i

T=

0 1
1 1

w = S 1 (T ( z ) ) =

iz + ( 1 + i ) z 1 i
=
z 1 + i
iz + (1 + i )

267

Example 6

Sol:
D.E.

Solve the Dirichlet problem in the figure using conform mapping by


constructing a linear fractional transformation that maps the given region into
the upper half-plane.
( 1 ) = 0
( 1 ) = 0
2
2
( x, y ) = 0
+
x 2 y 2

1 1
B.C. ( 1 ) = 0, ( 2 ) = 1, 1 = z z = 1 , 2 = z z =
2 2
1 1
R = z z < 1, z >
2 2

Let 1 and 2 pass through zp = 1 (can be picked as a pole)


1 , 2 : circles

E
( 2 ) = 1

( 1 ) = 0

( 1 ) = 0

1 ', 2 ' : lines

(1) To find w = T ( z ) : A = i
B = 1
E =1

A ' = T (i ) = 0

B ' = T ( 1) = 1

E ' = T (1) =

( w w1 )( w2 w3 ) = ( z z1 )( z2 z3 )
( w w3 )( w2 w1 ) ( z z3 )( z2 z1 )

( w 0 )(1 ) = ( z i )( 1 1)
( w )(1 0 ) ( z 1)( 1 i )

268

w = T ( z ) = (1 i )

w = T ( z) : C =

z i
: A, B, E
z 1
1

1 1
+ i
2 2
D=0

A ', B ', E '

1 1
+ i = 1 + i
2 2
D ' = T ( 0) = 1 + i

( 2 ) = 1

E ' = T (1) =

( 1 ) = 0

2 '

2
2
+
( u, v ) = 0, ( u , v ) R '
u 2 v 2

D.E.

( 1 ) = 0

1 '

C'=T

E =1

(2) To solve

( 1 ) = 0

( 1 ) = 0

( 2 ') = 1

B.C. ( 1 ') = 0, ( 2 ') = 1

( u, v ) = v = Im w

(x
=

( 1 ') = 0

z i
x + iy i
= (1 i )
z 1
x + iy 1

u + iv = w = T ( z ) = (1 i )

+ y 2 2 x 2 y + 1) + i (1 x 2 y 2 )

( x 1)

+ y2
269

( x, y ) = ( w ) = (T ( z ) ) = Im w = v = Im (T ( z ) )

The level curves ( x, y ) =


c
x
1+ c

1
+y =
1+ c

(1 x

( x 1)

y2 )
2

+ y2

(1 x
=

( x 1)

y2 )
2

+ y2

=c

270

Contents
20.1 Complex Functions as Mappings
20.2 Conformal Mapping and the Dirichlet Problem
20.3 Linear Fractional Transformations
20.4 Schwarz-Christoffel Transformations

271

Mapping to Polygonal Regions


Riemann mapping theorem
1. D = { z z < 1} : open disk
2. D ' : simply connected domain
conformal mapping onto
analytic function g : D

z -plane

D'

D'

w-plane
272

D = { z = x + iy y > 0} : upper half-plane

w = h ( ) = h ( g ( z ) )

D'

conformal mapping

z -plane

w-plane

Linear fractional
transformation
=

Riemann mapping
theorem

az + b
= g ( z)
cz + d

w = h ( ) analytic

D ''

D '' = < 1

-plane

Note: pure existence theory, does not tell how to find the mapping
273

Schwarz-Christoffel transformation w = f ( z )
D : upper half plane

D':

bounded
unbounded

polygonal region

Simply connected domain


D = { z = x + iy y > 0} : upper half-plane

w4

w5
D

z -plane

4
D'

D'

1
w1

2
w2

w3

1
w1

w4

w3

w2

274

dwa

wa

D'

Special Cases

dza

w = f ( z ) = ( z x1 )

dzb

za

0 < < 2

= z x1

x1

zb

(1 )

dwb
wb

w = = ( z x1 )

translation

real power function

rei = w = = ( ei ) = ei

:0 ~

= : 0 ~

dw
1
= f ' ( z ) = ( z x1 )
dz

D ''

dw = ( z x1 )

dz

arg dw = arg + ( 1) arg ( z x1 ) + arg dz

Note: zb > x1, and za < x1

arg dwa = arg + ( 1) arg ( za x1 ) + arg dza

As z moves from za to x1 and then to zb,


w moves from wa to 0 and then to wb,
arg dw changes by (1).

arg dwb = arg + ( 1) arg ( zb x1 ) + arg dzb

arg dwb arg dwa = ( 1) arg ( zb x1 ) arg ( za x1 ) + ( arg dzb arg dza )
= ( 1)( 0 ) + ( 0 0 )
= (1 )

275

w = f ( z ) analytic function
dw
( 1)
( 1)
= f ' ( z ) = A ( z x1 ) 1 ( z x2 ) 2 , x1 < x2
dz

( z x2 )( ) dz
arg dw = arg A + (1 1) arg ( z x1 ) + ( 2 1) arg ( z x2 ) + arg dz
dw = A ( z x1 )

(1 1)

2 1

arg dwb arg dwa = (1 1) arg ( zb x1 ) arg ( za x1 ) + ( 2 1) arg ( zb x2 ) arg ( za x2 )


= (1 1)( 0 ) + ( 2 1)( ) = (1 1 )
arg dwc arg dwb = (1 1) arg ( zc x1 ) arg ( zb x1 ) + ( 2 1) arg ( zc x2 ) arg ( zb x2 )
= (1 1)( 0 0 ) + ( 2 1)( 0 ) = (1 2 )
zc

dwc

(1 2 )
2

dzb

dza

za

D'

x1

zb

dzc
x2

zc

dwb

dwa
wa

w2

wb
(1 1 )

w1
276

THEOREM 20.4

Schwarz-Christoffel Formula

D = { z Im z > 0} : upper half plane

P can be bounded or unbounded.


(j+j) = , or j + j = 1.

P = { z Im z = 0} : boundary of D

P ' : polygon in w-plane with vertices w1 , w2 ,..., wn


interior angles 1 , 2 , ..., n

f ( z ) : analytic function in D
f ' ( z ) = A ( z x1 )
= A ( z x1 )

(1 1)

( z x2 )(

( z x2 )

2 1)

... ( z xn )

... ( z xn )

( n 1)

(
(z x )
n

j 1)

dz + B = A

j =1

conformal mapping: D
P

= A ( z x j )

( j 1)

j =1

= A ( z x j )

j =1

0<j < 2

x1 < x2 < ... < xn


w = f ( z) = A

exterior angles 1 , 2 , ..., n

(z x )

dz + B

j =1

D'
P' (not conformal)

x1 , x2 ,..., xn P

w1 , w2 ,..., wn P '
277

(1 n )
= n

n
P'
D'

wn

(1 )
j

= j
wj

x1

x2

xj

xn

1
w1

(1 1 )

(1 2 )

w2

278

Note:
(1) Judicious choice of three xj can simplify the computation of f (z), and the remaining
points depend on the shape of targeted polygon.
(
( z x )
n

(2) w = f ( z ) = A

j 1)

( z x )

dz + B = A

j =1

dz + B

j =1

= Ag ( z ) + B = A + B
g ( z) =

( z x )

( j 1)

P '' (interior of polygon)

Conformal mapping: P

dz

j =1

w = A + B Linear function: P ''

P ' (desired target polygon)

(3) D ' : bounded polygonal region

(1 1 ) + ... + (1 n ) = 1 + ... + n = 2

1 + ... + n = n 2

1 + ... + n = 2

Only (n 1) of n interior angles should be included in the Schwarz-Christoffel formula


As z moves along P, it never passes over xn One of xj may be chosen at , say xn =
1
( z xn )( ) term may be dropped
n

( z x1 )(

w = f ( z) = A

( z x1 )

=A

1 1)

... ( z xn 1 )

... ( z xn 1 )

( n1 1)

n1

dz + B
279

dz + B

Example 1
Use the Schwarz-Christoffel formula to construct a conformal mapping from
the upper half-plane to the strip v 1, u 0
Sol:

Select x1 = 1, x2 = 1
w2 = f ( x2 ) = f (1) = i

w1 = f ( x1 ) = f ( 1) = i

1 =

, 2 =

f ' ( z ) = A ( z x1 )
=

(z

(1 1)

f ( z ) = Ai

1
2

1 = , 2 =

1)

1/2

2 1)

2 1/2

i = w2 = Ai sin
f ( z) =

2i

sin 1 z

1/2

( z 1)

1/2

Ai

2 1/2

w1

dz + B = Ai sin 1 z + B

i = w1 = Ai sin 1 ( 1) + B = Ai
1

= A ( z + 1)

(1 z )

(1 z )

( z x2 )(

1
2

(1) + B = Ai

check f ( i ) =

w2

+B
A=
+B
2i

sin 1 i = 0.5611

, B=0

280

Example 2
Use the Schwarz-Christoffel formula to construct a conformal mapping from
the upper half-plane to the region shown in the figure
Sol: Select x = 1, x = 1
1
2
w1 = f ( x1 ) = f ( 1) = ai

1 =

w2 = f ( x2 ) = f (1) = 0

, 2 =
2
2

f ' ( z ) = A ( z x1 )
=

3
2

1 = , 2 =

(1 1)

A ( z + 1)

(z

f (z) = A

1)
z

( z x2 )(

2 1)

1
2

= A ( z + 1)

1/2

1/2

1/2

(z

1)

1/2

(z

dz + B = A ( z 2 1)

1/2

1)

1/2

ai = w1 = A cosh 1 ( 1) + B = A i + B
0 = w2 = A cosh 1 1 + B = 0
a

w = f ( z) =

( z 1)

(z

1)

1/2

+ cosh 1 z

A=

+ cosh 1 z + B

, B=0

check, say pick z = i

w=?
281

Example 3
Use the Schwarz-Christoffel formula to construct a conformal mapping from the
upper half-plane to the interior of the equilateral triangle shown in the figure
Sol:

D ' : bounded polygonal region

Only two of three interior angles should be included


Select x1 = 0, x2 = 1, x3 =

1 = 2 = 3 =
f ' ( z ) = A ( z x1 )
=

(1 1)

A
z

2/3

f ( z) = A

( z 1)

2/3

z
0

w3 = f ( )

w2 = f (1) = 1

w1 = f ( 0 ) = 0

2/3

( s 1)

0 = w1 = f ( 0 ) = B

1 = 2 = 3 =

( z x2 )(

2 1)

1
3

= A ( z 0)

2/3

( z 1)

2/3

Only (n 1) of n interior
angles should be included in
the Schwarz-Christoffel
ds
+
B
formula
2/3
B=0

282

1 = w2 = f (1) = A

2/3

( s 1)

( z) =

B ( m, n ) =

t z 1e t dt

A=

2
3

1
1

3
3
2

2
i
3

e3
w = f ( z) =
1 1
B ,
3 3

ds = A

2/3

t m 1 (1 t )

n 1

dt =

( m) ( n)
(m + n)

Note: error of this example in textbook

e3
=
1 1
B ,
3 3
z

1
1

2
3
3 3 i
e
2

m > 0, n > 0

Re(z) > 0, (z+1) = z (z)

2/3

Beta function

Gamma function

ds = A s 2/3 (1 s )

2/3

2
i
3

( s 1)

2/3

check: w3 = exp(i/3) = f ( ) ??

ds

283

nonpolygonal region (

Upper half plane

polygonal region)

Example 4
Use the Schwarz-Christoffel formula to construct a conformal mapping from the
upper half-plane to the upper half-plane with the horizontal line v = , u 0 ,
deleted.
Sol:
Non-polygonal target region:
approximated by a polygonal region
D1 '

D'

D'

u0

Select x1 = 1, x2 = 0
w1 = f ( 1) = i

w2 = f ( 0 ) = u0

f ' ( z ) = A ( z x1 )
u0

(1 1)

( z x2 )

( 2 1)

= A ( z + 1)

1 1

w1 = i D1 '

z 2 1

1 2 , 2 0; 1 2, 2 0

f ' ( z ) = A ( z + 1) z 1 = A 1 +

1
z

w = f ( z ) = A ( z + Ln z ) + B

w 2 = u0

284

i = w1 = f ( 1) = A ( 1 + Ln ( 1) ) + B = A ( 1 + i ) + B
= w2 = f ( 0 ) = A ( 0 + Ln 0 ) + B =

A, B: complex numbers

can not obtain A and B here

w = f ( z ) = A ( z + Ln z ) + B

Observe the image of the upper half plane: g ( z ) = z + Ln z


g ( t ) = t + log e t + iArg t = u + iv

t : 1

For t > 0
v ( t ) = Arg t = 0
u ( t ) = t + log e t ~

t : 1 0
1

For t < 0
v ( t ) = Arg t =

shift

u ( t ) = t + log e t ~ 1

g ( t ) : positive x-axis

g ( t ) : upper half-plane
w = f ( z ) = ( z + Ln z ) + 1

u -axis

upper half-plane with the horizontal line v = , u 1 deleted


w = f ( z ) = ( z + Ln z ) + 1 is the solution by shifting
g ( z ) = z + Ln z for +1

285

Homework 9
20.3
3, 11, 19, 21, 18

20.4
1, 7, 12, 13

286

You might also like